Vous êtes sur la page 1sur 80

ACKNOWLEDGEMENT

The time and effort provided by the following individuals who served as members of this committee are
greatly appreciated:
James VanRhee, MS, PA-C. Project Director
Linda Allison, MPH, MD
Mark Archambault, MHS, RPA-C
Petar Breitinger, MPAS, PA-C
Christine Bruce, MHSA, PA-C
Ralph Rice, MPAS, PA-C
Eric Vangsnes, MSA, PA-C
Donna Yeisley, MEd, PA-C
Kim Cavanagh, MPAS, PA-C

DEDICATION
This examination would not have been possible without the years of commitment of the MR. TIB
Development Committee. Numerous PA educators from across the nation provided their experience and
insight as questions for MR. TIB. It has been this data bank that served as the building blocks for
PACKRAT.
APAP is proud to be able to continue in the tradition of quality fostered by the forerunners of the selfassessment examination for physical assistants. It is our honor to dedicate PACKRAT to:
Jesse C. Edwards, MS
Claire S. Parker, PhD
University of Nebraska, Physician Assistant Program

Copyright 2008. Physician Assistant Education Association

ASSOCIATION OF PHYSICIAN ASSISTANT PROGRAMS


Physician Assistant Clinical Knowledge Rating and Assessment Tool (PACKRAT)
Form 12
Directions and Explanations

TABLE OF CONTENTS

I.

Introduction

II.

Explanation of the Score Report

Page 1: Scores Your total Score and Group Comparisons


Page 2: Your strengths, Weaknesses, and Quality of Responses
Page 3: Your Individual and Correct Responses
Page 4: Your Responses by Task and Category
Page 5-6: Your Profile Comparison: Demographic Profile

2
2
2
2
2

III.

Recommendations for Using the Feedback Package

IV.

Study Resources

V.

Answer Key

VI.

Examination Explanations

VII.

Comment Form

101

Copyright 2008. Physician Assistant Education Association. All rights reserved. No part of this publication may be reproduced or
transmitted in any form or by any means, electronic or mechanical, including photocopy or recording, or any information and
retrieval system, without permission in writing from the Physician Assistant Education Association.

Copyright 2008. Physician Assistant Education Association

PHYSICIAN ASSISTANT CLINICAL KNOWLEDGE RATING


AND ASSESSMENT TOOL (PACKRAT)

I. Introduction
The Physician Assistant Clinical Knowledge Rating and Assessment Tool (PACKRAT) was developed by
a volunteer committee of experts and is based on the content outline of a nationally recognized
competency examination. The following is a description of the content of PACKRAT:

PACKRAT EXAMINATION MATRIX


PACKRAT EXAMINATION MATRIX
CONTENT AREA
NUMBER OF ITEMS
CONTENT AREA
NUMBER OF ITEMS
1. History & Physical
36
1. History
& Physical
26 32
2. Diagnostic
Studies
2. Diagnostic
Studies
36 41
3. Diagnosis
3. Diagnosis
45 22
4. Health Maintenance
4. Health
Maintenance
22 31
5. Clinical
Intervention
5. Clinical
Intervention
32 41
6. Clinical
Therapeutics
6. Clinical
Therapeutics
48 22
7. Scientific
Concepts
7. Scientific Concepts
16
TOTALS
225
TOTALS
225
Additionally, questions also apply to the following clinical specialties:
Additionally, questions also apply to the following clinical specialties:
A. Cardiology
J. Obstetrics/Gynecology
A. Cardiology
Neurology
B. Dermatology
K. I.Orthopedics/Rheumatology
B. Dermatology
Obstetrics/Gynecology
C. Endocrinology
M.J.Psychiatry/Behavioral
Medicine
C. Endocrinology
Orthopedics/Rheumatology
F. Gastrointestinal/Nutritional
N. K.
Pulmonology
D. ENT
Pediatrics
H. Hematology
P. L.
Urology/Renal
E. Ophthalmology
Psychiatry/Behavioral
Medicine
I. Neurology
Q.M.
Infectious
Diseases
F. Gastrointestinal/Nutritional
N. Pulmonology
G. Geriatrics
O. Surgery
H. Hematology
P. Urology/Renal

The task and specialty categories for each item are listed in the answer key on page 5; your feedback
package contains a breakdown of responses by the task and clinical specialty category. Pay particular
attention to the questions you answered incorrectly and determine the specialty for that question and use
this information to identify weaknesses.
The PACKRAT provides a detailed feedback report of performance and it is available to anyone at any
time. Explanations were developed for all the questions to provide a rationale for correct, as well as
incorrect, answers. This information will help determine strengths and weaknesses with respect to the
PACKRAT content outline. If you have weaknesses in specific areas, you may need to obtain additional
clinical experience in those areas.

Copyright 2008. Physician Assistant Education Association

This booklet is designed to explain and interpret the information contained in the accompanying
computerized score report. You can use the report package to learn more about your abilities.

II. Explanation of the Score Report


This section provides an interpretation of each
page of the computerized score report you
received. You should have your computer
score report in front of you. Begin on page 1
of the report and read the following
information.
Page 1: Scores Your Total Score and
Group Comparisons
Page 1 is an overview of the PACKRAT
feedback report. Toward the bottom of the
page is your examination score. This score
shows the number of questions you answered
correctly out of a possible 225. The average
score for all first-year and second-year
candidates who have taken the PACKRAT to
date is also given.
Page 2: Your Strengths, Weaknesses, and
Quality of Responses
Page 2 of the score report gives an overview
of the content area in which your performance
is categorized as Strong, Satisfactory, or
Needing Improvement. These areas are
based on the examination matrix on page 1.
In each content area, your answers have
been classified as correct, acceptable,
unsatisfactory, or harmful. A definition of
these classifications is also provided on this
page. Pay particular attention to the areas
under Needing Improvement, as these areas
should be noted for further study. Also check
the answer key for the specialty area of these
items. If you selected a harmful answer in any
content area, it will be automatically placed in
the
Needing
Improvement
category,
regardless of the number of correct answers
selected. Carefully review these questions
and their explanations and specialty
classifications in Section VI to help you
understand why your answers were correct.
Page 3: Your Individual and Correct
Responses

one, the proper response appears in


parentheses. Use Section VI with this page to

review the rationale for each option that is


provided in the explanations, which are
referenced to the study resources. The
explanations may help you understand why
one answer is more appropriate than another,
or not the best answer, and why some of your
answers may have been incorrect. If the
option you chose was judged potentially
harmful to the patient or others, an asterisk (*)
appears before your answer. Options
classified as potentially harmful may identify
serious weaknesses. Go over these questions
carefully and read the explanations for the
correct answers. You may be able to identify
areas where you need further study.
Page 4: Your Responses by Task and
Specialty Category
Page 4 lists your responses by both specialty
and task category. You will be bale to identify
the areas of the content outline where you
may have difficulty. The numbers reflect how
many items you answered correctly out of the
total possible correct within each task and
specialty area. Categories 1 through 7 identify
the task areas and A-P the clinically specialty
areas. If you missed a significant number of
items in an area, check the key and go over
the explanations for the items in these areas.
Page 5-6: Your Profile
Demographic Profile

Comparison:

Page 5 is the beginning of the Demographic


Profile Comparison for programs. This profile
shows the reported demographic information
and compares your students information to
the entire group of individuals who have taken
the PACKRAT to date. The demographic data
are based on the information your students
provided when answering the demographics
questions. The summary demographic
information shown reflects all the data
compiled for either first or second-year
students who taken the PACKRAT.

Page 3 lists your answers to all questions.


When your answer differs from the correct
Copyright 2008. Physician Assistant Education Association

RESPONDENT DEMOGRAPHIC
INFORMATION
C. Number of months of clinical rotations
completed at the time of this exam?

Your response:

(
(
(
(
(
(

)
)
)
)
)
)

1.
2.
3.
4.
5.
6.

All Second-year
Respondents:

None
Less than 3 months
3 to 6 months
7 to 9 months
10 to 12 months
Greater than 12 months

( 0% )
( 5% )
( 15%)
( 20%)
(45%)
(15%)

Programs can utilize this information in order


to compare the characteristics of their first
year students and second year students to the
national demographics of first year and
second year students whove taken the
PACKRAT test.
III. Recommendations for using The
Feedback Package
As a current physician assistant student,
PACKRAT can be a useful self-evaluation
tool. Through careful review of question
explanations, noting specific tasks and
content areas, you will be able to assess your
current strengths and weaknesses. You will
be able to identify particular areas in which to
concentrate more effort as you continue your
studies. By concentrating your effort on the
areas in which you did not do well, you may
improve your performance, and you may have
a better chance of passing the proctored
examination.
However,
APAP
cannot
guarantee that this will occur, since the
conditions under which you attempted the
PACKRAT may have been different from
those in a standardized administration of a
proctored examination.
Use the explanations in Section VI to analyze
why you chose various options. Again, pay
particular attention to the options that were
judged potentially harmful or unsatisfactory.
Look at the question and the four options
again to see why the answer you chose was
incorrect. If there appears to be a deficit in
your exposure to a particular clinical specialty,

Copyright 2008. Physician Assistant Education Association

perhaps further study would make you more


familiar with these situations.
Once you have completely reviewed your
score report and this booklet, APAP hopes
you will use this information to improve your
overall performance, either on the job or on
future certification examinations. Should you
wish to provide suggestions about PACKRAT
to APAP, you will find a comment form on the
last page of this booklet.
IV. Study Resources
A variety of textbooks are currently available
to assist candidates in preparing for the
certification examination. For additional
information, you may contact a faculty
member at an educational program or an
experienced colleague if you need help
determining which references to review in a
specific content area. A short list of general
textbooks is below. All examination questions
are related to material found in these
resources.
Please note that the books on this list are not
available from APAP. This is not intended as
an all-inclusive list, and the materials listed
below are suggested study materials only.
1.

2.

3.

4.

5.

6.

7.

Andreoli TE, et al (eds). Cecils


Essentials of Medicine. 5th ed.
Philadelphia, PA: WB Saunders
Co., 2001.
Ballweg R et al. Physician Assistant:
A Guide to Clinical Practice. 3rd ed.,
Saunders, 2003
Bickley, LS. Bates Guide to
Physical Examination and History
Taking. 9th ed. Philadelphia, PA: JB
Lippincott Co., 2007
Beckmann CR, et al. Obstetrics &
Gynecology. 5th ed. Philadelphia,
PA: Lippincott Co., 2006.
Behrman RE, et al. Nelsons
Textbook of Pediatrics. 17th ed.
Philadelphia, PA: WB Saunders Co.,
2000.
Berkowitz, C. Pediatrics: A Primary
Care Approach, 2nd ed.
Philadelphia, PA: WB Saunders,
2000.
Fauci AS, et al ( eds). Harrisons
Principles of Internal Medicine. 16th
ed. New York, NY: McGraw-Hill,
Inc., 2005.

8.

9.

10.

11.

12.

13.

14.

15.

16.

17.

18.

19.

20.

21.

DeCherney AH & Pernoll ML (eds.)


Current Obstetric & Gynecological
Diagnosis & Treatment, 9th ed.,
Norwalk, CT: Appleton & Lange,
2003
Ellsworth AJ, et al. (eds). Mosbys
Medical Drug Reference.
Philadelphia, PA: Mosby Yearbook,
Inc., 2005.
Wolf K, Johnson RA, and Surmond
D. Fitzpatricks Color Atlas and
Synopsis of Clinical Dermatology.
5th ed. New York, NY: McGraw-Hill,
Inc., 2005
Goldman J and Bennet JC. Cecil
Textbook of Medicine. 22nd .
Philadelphia, PA: WB Saunders
Co., 2004.
Goroll, AH, Mulley, AG & May, LA.
Primary Care Medicine: Office
Evaluation and Management of the
Adult Patient. 4th Ed., Lippincott
Williams & Wilkins.
Hay WW, et al. Current Pediatric
Diagnosis and Treatment. 17th ed.
Norwalk, CT: McGraw Hill, 2005.
Kaplan HI and Sadock BJ (eds).
Synopsis of Psychiatry. 9th ed.,
Philadelphia, PA: Williams &
Wilkins, 2003.
Katzung BG. Basic and Clinical
Pharmacology. 9th ed. Stamford,
CT: Appleton & Lange, 2004.
Mandel GL, Bennett JE, and Dolin
R. Principles and Practice of
Infectious Disease. 6th ed.,
Churchill Livingston, 2005.
Marx, J., Hockberger, RS, and
Walls, RM. Rosen s Emergency
Medicine: Concepts and Clinical
Practice. 6th ed., Philadelphia, PA:
Churchill Livingston, an Elsevier
company, 2005.
McPhee SJ, et al. Pathophysiology
of Disease. 4th ed., Mcgraw Hill,
2002.
Mercier LR, et al. Practical
Orthopedics. 5th ed. Philadelphia,
PA: Mosby Yearbook, Inc., 2000.
Mettler FA, et al. Primary Care
Radiology. Philadelphia, PA: WB
Saunders, Co., 2000.
Howland RD nad Mycek MJ.
Lippincotts Illustrated Reviews:
Pharmacology. 3rd ed. Baltimore,
MD: Williams & Wilkins, 2005.

Copyright 2008. Physician Assistant Education Association

22.

23.

24.

25.

26.

27.

28.

29.

30.

31.

32.

Noble J, et al. Textbook of Primary


Care Medicine. 3rd ed. Philadelphia,
PA: Mosby, 2001.
Riordan-Eva, P. and Whitcher, JP.
Vaughn & Ashburys General
Opthalmology. 16th ed. McGraw Hill,
2003.
Sacher RA and McPherson RA.
Widmanns Clinical Interpretation of
Laboratory Tests. 11th ed. FA Davis
CO., 2000
Schwartz SI, et al. Principles of
Surgery. 8th ed. New York, NY:
McGraw-Hill, Inc. 2004.
Skinner HB (ed) Current Diagnosis
& Treatment in Orthopedics. 3rd ed.
Norwalk, CT: Appleton & Lange,
2003.
Steinberg GG. Orthopedics in
Primary Care. 3rd ed. Philadelphia,
PA: Lippincott Williams & Wilkins,
3rd ed, 1999.
Tierney LM, et al. Current, Medical
Diagnosis and Treatment. 45th ed.
Stamford, CT: Appleton & Lange,
2003.
Tintinalli JE, Kelen GD, and
Stapezynski JS. Emergency
Medicine: A Comprehensive Study
Guide. 6th ed. New York, NY
McGraw-Hill, Inc., 2004.
Townsend CM. Sabistons Textbook
of Surgery. The Biological Basis of
Modern Surgical Practice. 17th ed.
Philadelphia, PA. WB Saunders, an
Elsevier company, 2004.
Doherty GM. Current Surgical
Diagnosis and Treatment. 12th ed.,
McGraw Hill, 2005.
Wilson WR. Current Diagnosis and
Treatment in Infectious Disease.
Norwalk, CT: Appleton & Lange,
2001.

THIS PAGE INTENTIONALLY


LEFT BLANK FOR ANSWER
KEY

Copyright 2008. Physician Assistant Education Association

ASSOCIATION OF PHYSICIAN ASSISTANT PROGRAMS


Physician Assistant Clinical Knowledge Rating and Assessment Tool
(PACKRAT) Form 12

EXPLANATIONS

1. Clinical Intervention/Pulmonology
Which of the following is a major contraindication to curative surgical resection of a lung tumor?
A. Liver metastases
B. Vagus nerve involvement
C. Non-malignant pleural effusion
D. Chest wall invasion of the tumor
Explanations
(c) A. Distant metastases, except for solitary brain and adrenal metastases are an absolute contraindication for
pulmonary resection. Other absolute contraindications include MI within past 3 months, superior vena cava syndrome
due to metastatic tumor, bilateral endobronchial tumor, contralateral lymph node metastases and malignant pleural
effusion.
(u) B. See A for explanation.
(u) C. See A for explanation.
(u) D. See A for explanation.
Ref: (31)

2. Diagnostic Studies/Infectious Diseases


A 23 year-old female complains of fever and genital pain. Examination reveals the presence of lymphadenopathy in
the groin and the presence of vulvar vesicles surrounded by erythematous skin. The diagnosis may be confirmed by
A. A culture of a vesicle using blood agar medium.
B. The presence of similar lesions on the fingers and hands.
C. A Gram stain of a scraping from the base of the lesion.
D. The presence of giant multinucleated cells on Tzanck smear.
Explanations
(u) A. Herpes virus will not grow on blood agar medium.
(u) B. Herpes genitalis occurs in skin that comes into contact with the herpes virus. Usually the infection is localized.
(u) C. A Gram stain will not identify the herpes virus.
(c) D. A Tzanck smear is the standard laboratory test to confirm the herpes virus as an etiologic agent of a vesicular
lesion on an erythematous base.
Ref: (32)

3. Clinical Intervention/Obstetrics/Gynecology
A 28 year-old woman is complaining of heavy uterine bleeding and pelvic pressure that has progressively worsened
over the past year. Evaluation reveals multiple moderate-sized uterine fibroids. The patient desires to have more
children. The most appropriate definitive treatment is
A. Myomectomy.
B. Hysterectomy.
C. GnRH agonists.
D. Oral progesterone.
Explanations
(c) A. Myomectomy is the definitive treatment of choice for moderate-sized uterine fibroids in young women who
desire to maintain reproductive capability.
(u) B. Hysterectomy is indicated as definitive treatment in a patient who does not desire to maintain reproductive
capability.
(u) C. GnRH agonists are used as an adjunct to surgery for treatment of uterine fibroids. Used alone, they would not
be considered definitive treatment.
(u) D. Oral progesterone may be used to suppress menorrhagia preoperatively. Used alone, it would not be
considered definitive treatment.
Ref: (4)

4. Clinical Intervention/Pulmonology
A 59 year-old otherwise healthy female develops acute dyspnea and chest pain one week post total abdominal
hysterectomy. Echocardiogram demonstrates normal heart size with normal right and left ventricular function. Lung
scan demonstrates two segmental perfusion defects. Which of the following is the next step in the management of
this patient?
A. Anticoagulation
B. Embolectomy
C. Thrombolysis
D. Inferior vena cava filter
Explanations
(c) A. Anticoagulation is the treatment of choice in patients with pulmonary embolism with normal ventricular function
and no absolute contraindications.
(u) B. Embolectomy is not indicated as initial treatment of a pulmonary embolism in patients with normal ventricular
function.
(h) C. Thrombolysis is contraindicated in patients within 10 days of having major surgery.
(u) D. An inferior vena cava filter is considered in patients with contraindications to anticoagulation therapy or failed
anticoagulation therapy.
Ref: (7)

5. Clinical Therapeutics/Cardiology
Long term use of which of the following drugs may cause a drug-induced lupus-type eruption?
A. prednisone
B. tetracycline
C. procainamide
D. oral contraceptives
Explanations
(u) A. Prednisone is not implicated in drug-induced skin reactions.
(u) B. Tetracycline and sulfonamides are known to cause a photosensitive rash on sun exposed areas of the skin.
(c) C. Procainamide and hydralazine are the most common drugs that may cause a lupus-like eruption.
(u) D. Oral contraceptives may induce erythema nodosum.
Ref: (11)

6. History & Physical/Pulmonology


Which of the following clinical manifestations is most commonly seen in viral croup?
A. drooling
B. wheezing
C. sputum production
D. inspiratory stridor
Explanations
(u) A. Drooling is common in epiglottitis not viral croup.
(u) B. Wheezing is noted in asthma.
(u) C. Sputum production is noted in bacterial infections.
(c) D. Viral croup typically presents with a barking cough and stridor.
Ref: (13)

7. History & Physical/ENT/Ophthalmology


Whispered voice test on a patient reveals decreased hearing in the left ear. Which of the following would be most
consistent with conductive hearing loss in the left ear?
A. Sounds best heard in the left ear on Weber test.
B. Air conduction longer than bone conduction in the left ear on Rinne test.
C. Sound best heard in the right ear on Weber test.
D. Bone conduction longer than air conduction in the right ear.
Explanations
(c) A. Sound best heard in the ear with decreased hearing on Weber test (in this case, the left ear) is indicative of
conductive hearing loss.
(u) B. With conductive hearing loss, bone conduction should be heard as long as or longer than air conduction of
sound in the effected ear. Air conduction lasting longer than bone conduction of sound would indicate sensorineural
hearing loss.
(u) C. Sound best heard in the ear with unaffected hearing on Weber test (in this case, the right ear) is indicative of
sensorineural hearing loss.
(u) D. With conductive hearing loss, bone conduction should be heard as long as or longer than air conduction of
sound in the effected ear. The right ear showed normal hearing on physical exam.
Ref: (3)

8. Diagnosis/Dermatology
A 65 year-old male presents with multiple lesions on his back. He denies any pruritis. Physical examination reveals
the presence of multiple scattered brown plaques with a raised, warty surface that appear to be stuck onto the skin
and feel greasy. Which of the following is the most likely diagnosis?
A. lentigines
B. actinic keratosis
C. keratoacanthomas
D. seborrheic keratosis
Explanations
(u) A. Lentigines most commonly are seen on the dorsum of the hand and appear as flat brown spots, often with
sharp borders.
(u) B. Actinic keratosis usually present as small patches of flesh-colored, pink or yellow-brown lesions often with an
erythematous component. The lesions are better felt than seen, having a rough, sandpaper feel and are often tender
to palpation.
(u) C. Keratoacanthomas usually occur as an isolated lesion on the face appearing as an erythematous, domeshaped nodule with a central keratinaceous plug.
(c) D. Seborrheic keratosis is a common benign plaque in the elderly that characteristically has a velvety or warty
surface associated with a stuck on appearance and greasy feel.
Ref: (10)

10

9. Diagnosis/Pulmonology
You are called to the nursery to see a male infant, born by uncomplicated vaginal delivery. He weighs 2,600 grams
and has one deep crease on the anterior third of each foot. Respirations are 88 breaths/minute with expiratory
grunting and intercostals retractions. He is cyanotic on room air and becomes pink when placed on 60% oxygen.
Chest x-ray shows atelectasis with air bronchograms. Which of the following is the most likely diagnosis?
A. neonatal pneumonia
B. congenital heart disease
C. hyaline membrane disease
D. chronic lung disease of prematurity
Explanations
(u) A. While tachypnea, grunting, retractions and cyanosis may be signs of neonatal pneumonia, they are primarily
late findings of progressive respiratory distress and would not be seen immediately at the time of delivery. A chest xray in pneumonia would also most commonly reveal an infiltrate or effusion.
(u) B. While congenital heart disease may present with cyanosis, the chest x-ray will reveal a cardiac abnormality,
such as cardiomegaly.
(c) C. Hyaline membrane disease is the most common cause of respiratory distress in the premature infant. The
infant typically presents with tachypnea, cyanosis and expiratory grunting. A chest x-ray reveals hypoexpansion and
air bronchograms.
(u) D. Chronic lung disease of prematurity is a complication in about 20% of infants with hyaline membrane disease.
It is defined as respiratory symptoms, oxygen requirement and chest x-ray abnormalities at 1 month of age so it
cannot be diagnosed at this time in this newborn.
Ref: (5)

10. Diagnosis/Psychiatry/Behavioral Medicine


A 54 year-old patient has acute onset of palpitations, tremulousness, profuse sweating, shortness of breath, and
numbness and tingling of the extremities. Physical examination reveals a pulse of 104 beats/min and regular,
respiratory rate of 30/min, blood pressure of 160/95 mm Hg. Arterial blood gases reveal a low pCO2. ECG shows no
acute changes. The most likely diagnosis is
A. anxiety neurosis.
B. atrial fibrillation.
C. depressive neurosis.
D. myocardial infarction.
Explanations
(c) A. Anxiety is characterized by short-lived, recurrent, unpredictable episodes of intense anxiety accompanied by
marked physiological manifestations.
(u) B. Atrial fibrillation would not account for abnormal respiratory rate or blood gases, and the pulse would be
irregular.
(u) C. Depressive neurosis may be associated with somatic complaints, but it is not manifested by an acute, abrupt
onset of them.
(u) D. Even though the patient manifests some signs of a myocardial infarction (sweating and numbness), the ECG is
normal.
Ref: (14)

11. Diagnosis/Urology/Renal
Lab results for a post-operative oliguric patient reveals an increased BUN to creatinine ratio. The patient has a low
fractional excretion of sodium (less than 1%). Which of the following is the most likely diagnosis?
A. prerenal azotemia
B. acute tubular necrosis
C. acute glomerulonephritis
D. obstructive uropathy
Explanations

11

(c) A. Patients who have prerenal azotemia with otherwise normal kidneys will have severe sodium retention in order
to help to save fluid. The amount of sodium in the urine is therefore very low.
(u) B. Acute tubular necrosis may occur in the post-operative setting but these kidneys are damaged and unable to
save sodium.
(u) C. Acute glomerulonephritis is a complication of a streptococcal infection wherein the immune complexes damage
the glomeruli and lead to hematuria, red blood cell casts, and proteinuria.
(u) D. Although patients who undergo abdominal surgery are at risk for damage to the genital urinary system, these
patients will not have sodium retention because it is a post renal, not a prerenal injury.
Ref: (28)

12. Clinical Therapeutics/Orthopedics/Rheumatology


Which of the following is the treatment of choice for stage one Lyme disease in a patient less than 12 years of age?
A. doxycycline
B. amoxicillin
C. chloramphenicol
D. azithromycin
Explanations
(u) A. See B for explanation.
(c) B. Amoxicillin is first line therapy in a patient less than 12 years of age due to the harmful effects of doxycycline on
teeth and bones in children.
(u) C. Chloramphenicol is only indicated in disseminated disease with known drug resistance. It is not indicated as
first line therapy due to its tendency to cause aplastic anemia.
(u) D. Azithromycin is not as effective as doxycycline or amoxicillin.
Ref: (13)

13. Diagnosis/Urology/Renal
A 52 year-old patient presents with fatigue, complaints of paleness, anorexia, nausea, and weight loss. The patient
also complains of numbness in his hands and feet and a recent occurrence of foot drop. He has a past history of
diabetes and hypertension. Based on his clinical presentation, which of the following disorders is most likely to be
responsible for this clinical picture?
A. chronic renal failure
B. middle cerebral artery occlusion
C. Guillain-Barre syndrome
D. Raynaud's phenomena
Explanations
(c) A. Patients with chronic renal failure will have accelerated atherosclerosis, hypertension, anemia due to lack of
erythropoietin production, a tendency toward GI symptoms of anorexia, nausea, and weight loss, and neurological
symptoms of peripheral neuropathy that occurs in a stocking and glove distribution along with peripheral motor
impairment such as foot drop and restless legs syndrome. Diabetes and hypertension are risk factors for the
development of chronic renal disease.
(u) B. Middle cerebral artery occlusion would consist of contralateral neurologic impairment rather than bilateral
stocking glove neurological impairment
(u) C. Guillain-Barre is a condition manifested by an ascending paralysis.
(u) D. Raynaud's phenomena is a clinical condition in which patients exhibit pain and pallor of their distal extremities
when they are exposed to cold. Upon rewarming, the patient will experience redness as part of the color changing
sequence. Raynaud's typically does not cause the other symptoms described in the case scenario.
Ref: (1)

12

14. Diagnostic Studies/Hematology


Which of the following leukemias is most closely associated with the Philadelphia chromosome?
A. hairy cell
B. acute lymphocytic
C. acute myelocytic
D. chronic myelocytic
Explanations
(u) A. Hairy cell leukemia is a rare lymphocytic leukemia. See D for explanation.
(u) B. See D for explanation.
(u) C. See D for explanation.
(c) D. Philadelphia chromosome is noted most commonly in patients with chronic myelogenous leukemia.
Ref: (11)

15. Scientific Concepts/Gastrointestinal/Nutritional


A 53 year-old man who had previously been in good health presents to the emergency department with a history of
passing approximately 300 cc of bright red blood from the rectum 45 minutes ago. From which of the following
anatomical sites is the bleeding most likely originating?
A. rectosigmoid
B. duodenum
C. stomach
D. esophagus
Explanations
(c) A. Rectosigmoid is the most common site leading to hematochezia.
(u) B. See A for explanation.
(u) C. See A for explanation.
(u) D. See A for explanation.
Ref: (11)

16. Health Maintenance/Orthopedics/Rheumatology


Which of the following treatment strategies is most likely to promote optimal health and limit the morbidity and
mortality associated with rheumatoid arthritis?
A. meningococcal vaccination
B. NSAIDs
C. methotrexate
D. corticosteroids
Explanations
(u) A. RA patients are at increased risk of infection and should receive influenza and pneumococcal vaccines, not
meningococcal.
(u) B. NSAIDs provide only symptomatic therapy and do not stop the disease progression of RA.
(c) C. Methotrexate is the most commonly prescribed DMARD for RA and has been proven to induce remission in
some patients.
(u) D. Chronic corticosteroid use is associated with greater toxicity than DMARDs and should be used only for
symptomatic therapy.
Ref: (11)

13

17. Clinical Therapeutics/Psychiatry/Behavioral Medicine


A 20 year-old female presents with episodes of binge eating, overuse of laxatives, and periods of starvation. Which of
the following is the best treatment option for this patient?
A. fluoxetine (Prozac)
B. gabapentin (Neurontin)
C. amitriptyline (Elavil)
D. phenelzine (Nardil)
Explanations
(c) A. Fluoxetine, a SSRI, is the drug of choice for the treatment of bulimia nervosa.
(u) B. Gabapentin is used in the treatment of seizure disorders.
(u) C. Amitriptyline, a tricyclic antidepressant, is not effective in the treatment of bulimia nervosa.
(u) D. Phenelzine, a monoamine oxidase inhibitor, is not indicated in the treatment of bulimia nervosa.
Ref: (28)

18. Diagnosis/Gastrointestinal/Nutritional
A 40 year-old female complains of acute right upper quadrant pain radiating to the back and low grade fever.
Laboratory evaluation indicates the presence of urinary bilirubin and an elevation of serum alkaline phosphatase.
Which of the following is the most likely diagnosis?
A. cholecystitis
B. viral hepatitis
C. Gilbert's syndrome
D. Dubin-Johnson syndrome
Explanations
(c) A. The presence of urinary bilirubin indicating conjugated hyperbilirubinemia coupled with the elevation of serum
alkaline phosphatase suggests biliary obstruction that may lead to cholecystitis.
(u) B. Although viral hepatitis presents with conjugated hyperbilirubinemia, aminotransferase elevation would
predominate, not alkaline phosphatase.
(u) C. Gilbert's syndrome presents as unconjugated hyperbilirubinemia so urinary bilirubin would be absent.
(u) D. Although this hereditary disorder presents with conjugated hyperbilirubinemia, liver enzymes would not be
elevated.
Ref: (11)

19. Scientific Concepts/Cardiology


Which of the following is a cause of high output heart failure?
A. myocardial ischemia
B. complete heart block
C. aortic stenosis
D. thyrotoxicosis
Explanations
(u) A. Low output heart failure occurs secondary to ischemic heart disease, hypertension, dilated cardiomyopathy,
valvular and pericardial disease, and arrhythmia.
(u) B. See A for explanation.
(u) C. See A for explanation.
(c) D. High output heart failure occurs in patients with reduced systemic vascular resistance. Examples include:
thyrotoxicosis, anemia, pregnancy, beriberi and Paget's disease. Patients with high output heart failure usually have
normal pump function, but it is not adequate to meet the high metabolic demands.
Ref: (28)

14

20. Diagnosis/Neurology
A 37 year-old male presents with headaches for the past 2 months. They occur daily and are worse in the morning. In
the past week, he has noticed a tendency to drop things from his right hand. On examination, vital signs are normal,
and general examination is unremarkable. Neurologic examination reveals mild weakness of the right upper extremity
compared to the left. Which of the following is the most likely diagnosis?
A. focal seizure disorder
B. intracerebral neoplasm
C. transient ischemic attack
D. amyotrophic lateral sclerosis
Explanations
(u) A. Focal seizure disorder is not associated with morning headaches.
(c) B. Intracerebral neoplasms may present with headaches that are worse in the morning, with improvement during
the day. Focal motor or sensory loss depends upon the tumor's location.
(u) C. A TIA is characterized by focal findings that resolve completely and spontaneously within 24 hours.
(u) D. Amyotrophic lateral sclerosis is a degenerative disorder characterized by motor weakness, but is not
associated with morning headaches.
Ref: (11)

21. History & Physical/Gastrointestinal/Nutritional


Which of the following signs is positive in the presence of ascites?
A. shifting dullness
B. CVA tenderness
C. ballottment sign
D. cutaneous hypersensitivity
Explanations
(c) A. Shifting dullness or the presence of a fluid wave is noted in patients with ascites.
(u) B. CVA tenderness is noted in patients with pyelonephritis.
(u) C. Ballottment test is used to assess a floating mass in the abdomen.
(u) D. Cutaneous hypersensitivity is used to identify a specific zone of peritoneal irritation.
Ref: (3)

22. Clinical Therapeutics/Pulmonology


A 42 year-old male with unremarkable past medical history is admitted to the general medical ward with communityacquired pneumonia. He has a 20 pack-year history of cigarette smoking. He is empirically started on ceftriaxone
(Rocephin). Which of the following antibiotics would be most appropriate to add to his empiric treatment regimen?
A. piperacillin (Pipracil)
B. vancomycin (Vancocin)
C. clindamycin (Cleocin)
D. azithromycin (Zithromax)
Explanations
(u) A. Antipseudomonal penicillins, such as pipercillin, added to an aminoglycoside or fluoroquinolone are indicated
for empiric treatment of nosocomial pneumonia.
(u) B. Use of vancomycin should be restricted to cases where there is a high index of suspicion of involvement of
methicillin resistant Staphylococcus aureus (MRSA).
(u) C. Clindamycin is indicated in cases of suspected anaerobe involvement.
(c) D. Patients with community-acquired pneumonia who require hospitalization on the general medical ward are
treated with an extended-spectrum beta-lactam antibiotic, such as ceftriaxone, with a macrolide, such as
azithromycin. Addition of a macrolide is also recommended due to the patient's smoking history and possible
involvement of Hemophilus influenzae.
Ref: (28)

15

23. Diagnosis/Neurology
During an influenza epidemic, a 6 year-old child is seen with fever and a severe sore throat. A throat swab is taken
for culture and the child is sent home. The next day, he is reported to have persistent vomiting and increased
lethargy. On examination, he is delirious and disoriented. No rash is noted. His reflexes are hyperactive. The liver
edge is 3 cm below the right costal margin in the midclavicular line. Which of the following is the most likely
diagnosis?
A. acute bacterial meningitis
B. Guillain Barr syndrome
C. Reye syndrome
D. measles encephalitis
Explanations
(u) A. Acute bacterial meningitis presents with fever, malaise, and neck stiffness. The liver is typically normal.
(u) B. Guillain Barr syndrome typically presents post-Campylobacter enteritis. Signs and symptoms include an
ascending weakness. No hepatomegaly is noted and reflexes are diminished.
(c) C. Reye syndrome is typically post-influenza or URI. The patient develops lethargy, drowsiness, and vomiting.
Babinski reflex is positive and hyperreflexia is noted. The liver is normal or enlarged.
(u) D. Measles encephalitis typically presents one week after the measles rash with ataxia, vomiting, and seizures.
Ref: (5)

24. Diagnostic Studies/Psychiatry/Behavioral Medicine


Which of the following laboratory abnormalities is most commonly noted in bulimia nervosa?
A. glycosuria
B. hypokalemia
C. metabolic acidosis
D. hyperalbuminemia
Explanations
(u) A. Glycosuria is associated with diabetes mellitus, not bulimia.
(c) B. Episodes of binge eating are followed by purging in the bulimic patient. Vomiting and laxative abuse are the
most common methods of purging, leading to hypokalemia.
(u) C. A metabolic alkalosis may be noted if potassium losses from purging are great enough.
(u) D. Serum albumin levels may be normal or decreased.
Ref: (11)

25. Diagnosis/Obstetrics/Gynecology
A 22 year-old G0P0 asymptomatic female is seen for her yearly gynecologic examination. She denies performing
self-breast exams and any family history of breast or gynecologic cancers. On palpation of her left breast, a solitary 1
cm rubbery, firm, well-circumscribed, non-tender breast mass is noted. In review of her records, similar findings were
noted on last year's examination. Which of the following is the most likely diagnosis?
A. fibroadenoma
B. breast fat necrosis
C. fibrocystic changes
D. intraductal papilloma
Explanations
(c) A. Fibroadenoma of the breast most commonly is seen in young females. Typical characteristics include a solitary
1-3 cm firm, painless, freely movable mass that does not change with the menstrual cycle and are slow growing. Most
are found as an incidental finding on physical examination or during self-breast examination.
(u) B. Fat necrosis of the breast is rare. It is thought to result from trauma to the breast and occasionally is
accompanied by ecchymosis. If left untreated, the mass eventually disappears.
(u) C. Fibrocystic changes in the breast are most common in females between 30-50 years of age. The disorder is
characterized by painful, often multiple and usually bilateral masses that are associated with increased pain and size
of breast mass premenstrually.
(u) D. Intraductal papillomas are usually non-palpable. The patient presents with complaints of spontaneous onset of
bloody, serous, or cloudy nipple discharge.
Ref: (4)

16

26. Clinical Therapeutics/Pulmonology


A 25 year-old male with a history of asthma presents complaining of increasing episodes of evening and daytime
symptoms. He is on a short acting inhaled beta agonist prn. He is presently using his short acting beta agonist on a
daily basis. Which of the following is the most appropriate addition to this patient's regimen?
A. methylxanthine oxidase inhibitor
B. long acting beta agonist inhaler
C. leukotriene inhibitor
D. inhaled corticosteroid
Explanations
(u) A. Methylxanthine oxidase inhibitor preparations may have beneficial effects in some patients, but their value is
limited due to a narrow therapeutic window and modest efficacy.
(u) B. Long acting beta agonist inhalers should not be used in place of anti-inflammatory therapy.
(u) C. Leukotriene inhibitors are less desirable alternatives to inhaled corticosteroids.
(c) D. According to the stepwise approach for managing asthma by the National Asthma Education and Prevention
Program, inhaled corticosteroids are indicated for mild to moderate persistent asthma.
Ref: (7)

27. Health Maintenance/ENT/Ophthalmology


Which of the following is considered a risk factor for retinopathy of prematurity?
A. maternal rubella infection
B. maternal alcohol abuse
C. perinatal oxygen therapy
D. family history of retinal detachment
Explanations
(u) A. While maternal rubella infection is a risk factor for ocular disease in the newborn, it is not a specific risk for
retinopathy of prematurity.
(u) B. Maternal alcohol abuse is associated with the development of fetal alcohol syndrome, which includes
craniofacial abnormalities, but does not include increased risk for retinopathy of prematurity.
(c) C. Risk factors for retinopathy of prematurity include perinatal oxygen therapy, low birth weight, prematurity, and
sepsis.
(u) D. If retinopathy of prematurity is not treated, retinal detachment may occur causing blindness. A family history of
retinal detachment is not considered a risk factor for the development of retinopathy of prematurity.
Ref: (5)

28. Scientific Concepts/Endocrinology


Which hormone is responsible for producing 1,25-dihydroxyvitamin D (calcitriol) in the kidney?
A. TSH
B. ACTH
C. PTH
D. CRH
Explanations
(u) A. TSH is thyroid stimulating hormone and comes from the pituitary gland. It is responsible for the production of
T4 within the thyroid.
(u) B. ACTH is adrenocorticotropin hormone and comes from the pituitary gland. It is responsible for the production of
glucocorticoids within the adrenal gland.
(c) C. PTH is parathyroid hormone and comes from the parathyroid glands. It is responsible in making calcitriol which
is needed for calcium absorption in the gut. Under normal renal function, 25-hydroxyvitamin D is converted to 1,25dihydroxyvitamin D in the distal convoluted tubule.
(u) D. CRH is corticotropin releasing hormone and comes from the hypothalamus. It is the stimulator hormone to the
pituitary gland which ultimately is the precursor to ACTH.
Ref: (11)

17

29. Diagnostic Studies/Pulmonology


A 73 year-old obese female with a 20 pack year smoking history presents complaining of chronic productive cough.
She states that it has been occurring over the past 3 years more frequently November through February. Which of
the following pulmonary function test values would you expect to find decreased?
A. tidal volume
B. forced expiratory volume in 1 second/forced vital capacity
C. residual volume
D. total lung capacity
Explanations
(u) A. Tidal volume is usually unchanged, residual volume and total lung capacity are decreased with a restrictive
disease pattern. This scenario presents with bronchitis which is an obstructive disease.
(c) B. Forced expiratory volume in 1 second/forced vital capacity is decreased in obstructive lung diseases such as
bronchitis.
(u) C. See A for explanation.
(u) D. See A for explanation.
Ref: (7)

30. Diagnosis/Cardiology
A 46 year-old male with no past medical history presents complaining of chest pain for four hours. The patient admits
to feeling very poorly over the past two weeks with fever and upper respiratory symptoms. The patient denies
shortness of breath or diaphoresis. On examination the patient appears fatigued. Vital signs reveal a BP of 130/80,
HR 90 and regular, RR 14. The patient is afebrile. Labs reveal a Troponin I of 10.33 ug/L (0-0.4ug/L). Cardiac
catheterization shows normal coronary arteries and an ejection fraction of 40% with global hypokinesis. Which of the
following is the most likely diagnosis?
A. myocarditis
B. pericarditis
C. hypertrophic cardiomyopathy
D. coronary artery disease
Explanations
(c) A. Myocarditis often occurs secondary to acute viral illness and causes cardiac dysfunction. Patients will
commonly have a history of a recent febrile illness. Chest pain may mimic that of a myocardial infarction and
Troponin I levels maybe elevated in one-third of patients. Contractile dysfunction is seen on catheterization and/or
echocardiogram.
(u) B. Pericarditis does not typically cause ventricular dysfunction and cardiac enzymes are usually normal.
(u) C. Hypertrophic cardiomyopathy is associated with ventricular hypercontractility.
(u) D. This patient had normal coronary arteries on cardiac catheterization, no signs of coronary artery disease.
Ref: (28)

31. Scientific Concepts/Obstetrics/Gynecology


Which of the following is the most common underlying cause of early postpartum hemorrhage?
A. uterine atony
B. genital tract trauma
C. coagulation disorders
D. retained placental tissue
Explanations
(c) A. Uterine atony is the most common cause of postpartum hemorrhage. Predisposing factors include any
conditions that cause excessive uterine enlargement, abnormal labor or conditions that interfere with uterine
contraction.
(u) B. Genital tract trauma may lead to lacerations of the lower genital tract that cause postpartum hemorrhage,
however this is much less common than uterine atony.
(u) C. Coagulation disorders can cause postpartum hemorrhage, but are not the most common cause.
(u) D. Retained placental tissue only acccounts for 5-10% of postpartum hemorrhage and usually occurs later.
Ref: (4)

18

32. Clinical Intervention/Neurology


A 30 year-old male has a history of weakness without pain on the left side of his face for the past 4 days. Physical
examination of the face reveals unilateral weakness to the left side, but not complete paralysis. The left eye does not
close completely. The most appropriate initial treatment is to
A. begin physical therapy.
B. refer for surgical ablation.
C. reassure patient and provide patient education.
D. initiate high dose steroids and recheck in 24 hours.
Explanations
(u) A. Physical therapy is not indicated for Bell's palsy
(u) B. Surgery has not been shown to provide any significant benefit in the treatment of Bell's palsy.
(c) C. Bell's palsy is usually a self-limited condition and typically resolves within a few weeks.
(u) D. Corticosteroids may be used if paralysis is complete or there is significant pain, but are not first-line.
Ref: (11)

33. Clinical Therapeutics/Gastrointestinal/Nutritional


A patient presents with a concern about an upcoming trip. He states he is traveling to Mexico and has a question
about how to treat diarrhea should it develop. In addition to promoting fluid intake which of the following is
recommended?
A. kaolin-pectin
B. ciprofloxacin
C. simethicone
D. doxycycline
Explanations
(u) A. Kaolin-pectin is not indicated in the treatment of traveler's diarrhea.
(c) B. Ciprofloxacin is indicated in the treatment of traveler's diarrhea.
(u) C. Simethicone is used in the treatment of intestinal gas.
(u) D. Doxycycline is not indicated in the treatment of traveler's diarrhea due to increasing resistance.
Ref: (32)

34. Scientific Concepts/Pulmonology


Which of the following has been found to be an important reservoir in the transmission of pertussis?
A. mosquitoes
B. adult humans
C. domestic pets
D. white-tailed deer
Explanations
(u) A. While mosquitoes have been implemented in the spread of several infectious diseases, pertussis is not one of
them.
(c) B. Since neither the vaccine nor the disease itself provides lasting immunity against pertussis, adult humans are
an important reservoir in the transmission of pertussis.
(u) C. Contact with domestic pets has led to the development of upper and lower respiratory infections that are
caused by Bordetella bronchiseptica, but not pertussis.
(u) D. White-tailed deer are part of the transmission cycle for Lyme disease, not pertussis.
Ref: (28)

35. Diagnostic Studies/Dermatology


A 34 year-old female presents with complaints of a skin rash associated with a low-grade fever, malaise and
anorexia. She denies any pruritis. Physical examination reveals the presence of a diffuse maculopapular rash
involving the palms and soles associated with generalized lymphadenopathy. Which of the following will confirm the
suspected diagnosis?

19

A. VDRL
B. patch testing
C. acetowhitening
D. Gram stain and culture
Explanations
(c) A. This patient most likely has secondary syphilis. Serological tests, like the VDRL, are usually positive during this
stage with high titers.
(u) B. Patch testing is utilized to identify a causative agent and confirm a diagnosis of allergic contact dermatitis.
(u) C. Acetowhitening helps in the detection of subclinical penile or vulvar warts.
(u) D. Gram stain and culture should be performed on lesions suspected to have a bacterial origin.
Ref: (10)

36. Scientific Concepts/Orthopedics/Rheumatology


Which of the following leads to retropatellar pain?
A. increased Q angle
B. increased quadriceps tone and strength
C. Osgood-Schlatter Disease
D. hamstring stretching
Explanations
(c) A. Patients with an increased Q angle have more force directed laterally during knee flexion and are at greater risk
of patellofemoral pain syndrome.
(u) B. Quadriceps strengthening aids in restoring patellofemoral tracking.
(u) C. Osgood-Schlatter disease involves the tibial tubercle, not the patellofemoral joint.
(u) D. Hamstring stretching aids in restoring patellofemoral tracking.
Ref: (26)

37. Diagnostic Studies/ENT/Ophthalmology


Which of the following is most helpful in the diagnosis of a retropharyngeal abscess?
A. CBC with differential
B. fever and a muffled voice on examination
C. CT of the neck with contrast
D. history of a recent throat infection
Explanations
(u) A. A CBC with differential would identify an infection but not specifically a retropharyngeal abscess.
(u) B. The presence of fever and a muffled voice on physical exam is not specific for a retropharyngeal abscess.
(c) C. CT of the neck is considered the "gold standard" for the diagnosis of a retropharyngeal abscess.
(u) D. A recent throat infection is not specific for a retropharyngeal abscess.
Ref: (29)

38. History & Physical/Urology/Renal


Which of the following is the most likely physical examination finding in a patient with Peyronie's disease?
A. inflammation of the glans of the penis
B. foreskin that cannot be retracted
C. chancre on the shaft of the penis
D. fibrous band on lateral portion of the penis
Explanations
(u) A. Inflammation of the glans of the penis is noted in balanitis.
(u) B. A foreskin that cannot be retracted is seen in phimosis.
(u) C. A chancre on the shaft of the penis is noted in primary syphilis.
(c) D. Peyronie disease typically presents with fibrotic areas under the penile skin along with a history of penile
curvature during erection.
Ref: (28)

20

39. Clinical Therapeutics/Cardiology


Which of the following antihypertensive agents is considered to have both alpha- and beta-blocker activities?
A. carvedilol (Coreg)
B. hydralazine (Apresoline)
C. minoxidil (Loniten)
D. spironolactone (Aldactone)
Explanations
(c) A. Carvedilol has both alpha- and beta-blocker activities.
(u) B. Hydralazine and minoxidil are considered vasodilators.
(u) C. See B for explanation.
(u) D. Spironolactone is a potassium-sparing diuretic.
Ref: (15)

40. History & Physical/Neurology


The most frequent finding in a person presenting with a brain abscess is
A. nuchal rigidity.
B. headache.
C. seizures.
D. vomiting.
Explanations
(u) A. Nuchal rigidity occurs in approximately 35% of patients with a brain abscess.
(c) B. Headache occurs in over 70% of patients with a brain abscess.
(u) C. Seizures occur in approximately 35% of patients with a brain abscess.
(u) D. Vomiting occurs in approximately 35% of patients with a brain abscess.
Ref: (28)

41. Clinical Intervention/Endocrinology


Radioactive iodine is most successful in treating hyperthyroidism that results from
A. Grave's disease.
B. subacute thyroiditis.
C. Hashimoto's thyroiditis.
D. papillary thyroid carcinoma.
Explanations
(c) A. Radioactive iodine is an excellent method to destroy overactive thyroid tissue of Grave's disease.
(u) B. Radioactive iodine is ineffective in subacute thyroiditis due to the thyroid's low uptake of iodine.
(u) C. Radioiodine uptake is low in Hashimoto's thyroiditis and is often transient.
(u) D. Papillary thyroid carcinoma is a common thyroid malignancy and must be treated by a thyroidectomy.
Ref: (7)

42. Health Maintenance/ENT/Ophthalmology


Which of the following is the most common cause of blindness in adults under the age of 70 years in the USA?
A. hypertension
B. diabetes mellitus
C. macular degeneration
D. retinal artery occlusion
Explanations
(u) A. See B for explanation.
(c) B. Diabetic retinopathy is the leading cause of blindness in adults under the age of 70 in the US.
(u) C. See B for explanation.
(u) D. See B for explanation.
Ref: (7)

21

43. Clinical Intervention/Pulmonology


Which of the following is an indication for hospitalization in a patient who has acute bronchiolitis?
A. pulse oximetry of 94% on room air
B. children between 4-6 months of age
C. moderate tachypnea with feeding difficulties
D. hyperinflation and interstitial infiltrates on chest x-ray
Explanations
(u) A. A pulse oximetry reading of 94% on room air is equivalent to a PaO2 of approximately 80 mm Hg which
indicates the child is not in severe respiratory distress.
(u) B. Children less than 2 months of age require hospitalization.
(c) C. Indications for hospitalization include moderate tachypnea with feeding difficulties.
(u) D. Hyperinflation and interstitial infiltrates on chest x-ray are frequently seen with acute bronchiolitis and by
themselves are not an indication for hospitalization.
Ref: (17)

44. Health Maintenance/Neurology


Which of the following is a milestone usually achieved by a 15-month old infant?
A. walks alone
B. puts three words together
C. feeds self well with spoon
D. builds tower of seven cubes
Explanations
(c) A. A 15-month-old infant should be able to walk alone.
(u) B. The ability to put three words together, feed oneself well with a spoon and build a tower of seven cubes does
not occur until 24 months.
(u) C. See B for explanation.
(u) D. See B for explanation.
Ref: (5)

45. Health Maintenance/Gastrointestinal/Nutritional


At birth an infant weighs 8 pounds. The optimal weight for this infant at 1 year would be
A. 16 pounds.
B. 20 pounds.
C. 24 pounds.
D. 28 pounds.
Explanations
(u) A. See C for explanation.
(u) B. See C for explanation.
(c) C. An infants birth weight should double by 4-5 months of age and triple by 1 year of age.
(u) D. See C for explanation.
Ref: (3)

46. Diagnosis/Cardiology
A 12 year-old boy presents to the office with pain in his legs with activity gradually becoming worse over the past
month. He is unable to ride a bicycle with his friends due to the pain in his legs. Examination of the heart reveals an
ejection click and accentuation of the second heart sound. Femoral pulses are weak and delayed compared to the
brachial pulses. Blood pressure obtained in both arms is elevated. Chest x-ray reveals rib notching. Which of the
following is the most likely diagnosis?
A. abdominal aortic aneurysm
B. pheochromocytoma
C. coarctation of the aorta
D. thoracic outlet syndrome

22

Explanations
(u) A. Abdominal aortic aneurysm is usually asymptomatic until the patient has dissection or rupture. It is uncommon
in a child.
(u) B. Pheochromocytoma classically causes paroxysms of hypertension due to catecholamine release from the
adrenal medulla, but does not cause variations in blood pressure in the upper and lower extremities.
(c) C. Coarctation is a discrete or long segment of narrowing adjacent to the left subclavian artery. As a result of the
coarctation, systemic collaterals develop. X-ray findings occur from the dilated and pulsatile intercostal arteries and
the "3" is due to the coarctation site with proximal and distal dilations.
(u) D. Thoracic outlet syndrome occurs when the brachial plexus, subclavian artery, or subclavian vein becomes
compressed in the region of the thoracic outlet. It is the most common cause of acute arterial occlusion in the upper
extremity of adults under 40 years old.
Ref: (5)

47. History & Physical/Obstetrics/Gynecology


Which of the following is the earliest and most reliable clinical manifestation of pre-eclampsia?
A. onset of proteinuria
B. elevation of blood pressure
C. excessive weight gain and edema
D. headache and visual distrubances
Explanations
(u) A. While proteinuria is a defining diagnostic criterion of pre-eclampsia, it is a late finding and may not be present
even with eclampsia.
(c) B. Elevated blood pressure is the most important and reliable diagnostic criterion. It is seen early in the course of
pre-eclampsia and may occur suddenly.
(u) C. Since weight gain and edema are common occurrences during pregnancy, they are less reliable indicators of
pre-eclampsia. Most current sources no longer list them as diagnostic criterion for pre-eclampsia.
(u) D. Headache and visual disturbances indicate severe pre-eclampsia and are late findings that indicate a need to
deliver the fetus.
Ref: (8)

48. Diagnostic Studies/ENT/Ophthalmology


A 26 year-old male presents with headache, sinus pressure, and sinus congestion for over a month. He has a thick
nasal discharge in the mornings, but this improves as the day goes on. He is afebrile. On exam, there is tenderness
over the face. TMs have normal light reflex. Nasal mucosa reveals thick yellowish discharge. Neck is supple, without
lymphadenopathy. Which of the following is the diagnostic study of choice?
A. transillumination of sinuses
B. routine sinus films
C. CT scan of sinuses
D. nasal culture
Explanations
(u) A. Transillumination is used in the initial evaluation of chronic or acute sinusitis, but is not sensitive or specific.
(u) B. See C for explanation.
(c) C. CT scan is more sensitive than plain films for the diagnosis and management of chronic sinusitis, and is
considered the gold standard for sinus imaging.
(u) D. Nasal culture is not indicated in the evaluation of chronic sinus infections.
Ref: (28)

49. Clinical Therapeutics/Cardiology


According to the recent JNC VII guidelines, a 34 year-old male who has type 1 diabetes mellitus and hypertension
should be started on which type of antihypertensive agent?
A. beta-blocker
B. loop diuretic
C. ACE inhibitor
D. thiazide diuretic

23

Explanations
(u) A. Beta blockers could potentially be harmful in a patient with diabetes mellitus. Use a cardioselective betablocker to reduce the incidence of hypoglycemia.
(u) B. See C for explanation.
(c) C. ACE inhibitors are effective in young patients. They are capable of providing protection to the kidney especially
in diabetes mellitus.
(u) D. See C for explanation.
Ref: (28)

50. History & Physical/Urology/Renal


A male patient presents with hematuria. Upon further questioning the patient states that the hematuria occurs at the
end of his urinary stream. Which of the following is the most likely source of blood?
A. renal pelvis
B. bladder neck
C. anterior urethra
D. ureter
Explanations
(u) A. Total hematuria, blood throughout the urinary stream, suggests a bladder or upper urinary tract source.
(c) B. Terminal hematuria, blood at the end of the urinary stream, suggests a bladder neck or prostatic urethral
source.
(u) C. Presence of blood at the beginning of the urinary stream suggests an anterior (penile) urethral source.
(u) D. Hematuria from the kidneys or ureter may be present microscopically or throughout the stream.
Ref: (29)

51. Diagnostic Studies/Pulmonology


Which of the following is the most common radiographic presentation of lung abscess?
A. cavitation
B. pleural thickening
C. hilar mass
D. hyperinflation
Explanations
(c) A. Cavitation is seen with lung abscess or progressive primary tuberculosis.
(u) B. Pleural thickening is noted in mesothelioma.
(u) C. Hilar and mediastinal abnormalities are common on chest radiography in patients with lung cancer.
(u) D. Hyperinflation is the main clinical feature in emphysema.
Ref: (7)

52. Diagnostic Studies/Neurology


A patient involved in a minor motor vehicle crash is brought to the emergency department by a family member who
was riding in the car, but was unhurt. The family member states that the patient was unconscious for about 2 minutes,
but seems "okay" now. What diagnostic procedure would be most helpful in assessing this patient?
A. head CT scan
B. lumbar puncture
C. skull radiographs
D. electroencephalogram
Explanations
(c) A. A head CT scan would provide evidence of fractures and demonstrate intracranial hemorrhage and cerebral
edema if present.
(u) B. A lumbar puncture would not be immediately indicated, but if the patient worsens it might be a consideration.
(u) C. Skull radiographs would only reveal skull fracture and not any potentially lethal intracranial injuries.
(u) D. An electroencephalogram is not needed because there is no history of seizures.
Ref: (29)

24

53. Clinical Intervention/Gastrointestinal/Nutritional


A patient presents complaining of vague anal discomfort. On examination, the patient is noted to have a few small
external hemorrhoids and edema in the anal region. Which of the following is the most appropriate intervention?
A. proctoscopy followed by a hemorrhoidectomy
B. increased dietary fiber and sitz baths
C. hemorrhoidal banding
D. inject a sclerosing agent
Explanations
(u) A. Hemorrhoidectomy should be used for permanently prolapsed internal hemorrhoids.
(c) B. Most hemorrhoids respond well to conservative treatment such as fiber and sitz baths.
(u) C. Banding and injection of sclerosing agents are used if mild prolapse, enlargement, or intermittent bleeding is
present.
(u) D. See C for explanation.
Ref: (7)

54. Clinical Therapeutics/Endocrinology


While awaiting operative removal of pheochromocytoma, which of the following classes of medications are used for
control of hypertension?
A. alpha-adrenergic blocker
B. beta-adrenergic blocker
C. ACE inhibitor
D. diuretic
Explanations
(c) A. Alpha-adrenergic blockers are used preoperatively to control hypertension in a patient with pheochromocytoma
that occurs from unopposed alpha stimulation when the tumor is manipulated.
(u) B. Beta-adrenergic blockers are used to control tachycardia and arrhythmias, if present, after the hypertension
has been controlled.
(u) C. ACE inhibitors and diuretics have no role in the treatment of hypertension from pheochromocytoma.
(u) D. See C for explanation.
Ref: (28)

55. History & Physical/Pulmonology


A foreign body lodged in the trachea that is causing partial obstruction will most likely produce what physical
examination finding?
A. stridor
B. aphonia
C. inability to cough
D. progressive cyanosis
Explanations
(c) A. An inspiratory wheeze is called stridor, which indicates a partial obstruction of the trachea or larynx.
(u) B. Aphonia, inability to cough and progressive cyanosis are seen with complete obstruction of the trachea, not
partial obstruction.
(u) C. See B for explanation.
(u) D. See B for explanation.
Ref: (5)

56. History & Physical/Psychiatry/Behavioral Medicine


A 65 year-old patient has a long history of schizophrenia that is treated with phenothiazines. On an unrelated clinic
visit, the patient has difficulty sticking out her tongue, facial tics, increased blink frequency, and lip-smacking
behavior. These involuntary movements are most suggestive of

25

A. tardive dyskinesia.
B. Parkinson's disease.
C. Huntington's disease.
D. Tourette's syndrome.
Explanations
(c) A. Tardive dyskinesia is characterized by abnormal involuntary movements of the face, mouth, tongue, trunk, and
limbs and may develop after months or years of treatment with neuroleptic drugs.
(u) B. Infrequent blinking, tremor, rigidity, and bradykinesia are characteristic of Parkinsonism.
(u) C. Although part of the differential for involuntary movements, this disease has a positive family history and
usually appears by age 50.
(u) D. Facial motor tics are the most common manifestation of this disorder, but symptoms begin before age 21.
Ref: (15)

57. Clinical Intervention/ENT/Ophthalmology


Which of the following is a potential complication of a traumatic hyphema?
A. retinal detachment
B. glaucoma
C. cataract formation
D. chronic conjunctivitis
Explanations
(u) A. Retinal detachment may occur from trauma but not from a hyphema.
(c) B. If the trabecular network becomes obstructed from the hyphema then glaucoma may occur.
(u) C. Cataracts may be caused by increasing age (most commonly), toxins, systemic disease, smoking, and
hereditary, but not by hyphemas.
(u) D. Chronic conjunctivitis is not known to be associated with hyphemas.
Ref: (23)

58. Health Maintenance/Dermatology


According to the Advisory Committee on Immunization Practices, which of the following is the recommended age
range for the first or initial MMR (measles, mumps and rubella) vaccination?
A. 2-4 months
B. 12-15 months
C. 4-6 years
D. 11-12 year
Explanations
(h) A. MMR vaccination is not approved for use in infants below the age of 12 months.
(c) B. The recommended age range for the initial vaccination against MMR is 12-15 months.
(u) C. The second dose of the MMR vaccine is routinely recommended to be administered between 4-6 years of age.
(u) D. Any child not previously vaccinated by 11-12 years of age may receive the vaccine. This is considered to be a
"catch-up" immunization and is not the recommended age range for the initial vaccination.
Ref: (13)

59. Diagnostic Studies/Pulmonology


A 64 year-old female with a 50 pack year smoking history, presents with worsening dyspnea on exertion, a persistent
cough, and increasing oxygen requirement from 2 to 3 liters. She denies any cardiac history. What is the most likely
chest x-ray finding in this patient?
A. pulmonary vascular congestion
B. left lower lobe infiltrate
C. apical infiltrates
D. hyperinflation with bullae

26

Explanations
(u) A. Pulmonary vascular congestion represents congestive heart failure not COPD.
(u) B. Left lower lobe infiltrate represent an infectious process, such as pneumonia.
(u) C. Apical infiltrates represent an infectious process, such as tuberculosis.
(c) D. Hyperinflation with bullae is a consistent finding in patients with emphysema, such as this patient.
Ref: (7)

60. Clinical Intervention/Psychiatry/Behavioral Medicine


A 48 year-old alcoholic, whose last drink was 6 days ago, presents to the clinic complaining of palpitations and
intermittent abdominal pain. On physical examination, blood pressure is 170/110 mm Hg, pulse 124/min, respirations
22/min, and temperature 100.4 degrees F. The patient is agitated and excitable. Cardiovascular examination reveals
tachycardia without murmurs, gallops, or rubs, and is otherwise normal. Which of the following is the most
appropriate intervention?
A. start antihypertensive medication
B. begin a cardiac work-up for angina
C. admit to the hospital for alcohol withdrawal
D. consult social services for appropriate disposition
Explanations
(u) A. See C for explanation.
(u) B. See C for explanation.
(c) C. This patient is exhibiting the signs and symptoms of alcoholic withdrawal characterized by delirium, autonomic
hyperactivity, perceptual distortions, and fluctuating levels of psychomotor activity. Seizures are a common
occurrence prior to DTs, but the delirium may occur without preceding seizures. This is a medical emergency, and if
untreated, it has a mortality rate of 20%.
(u) D. See C for explanation.
Ref: (14)

61. Diagnostic Studies/Urology/Renal


A 20 year-old male presents with a hard mass on the testicle. There has been no previous infection or trauma to the
area. Which of the following is the initial diagnostic evaluation to pursue?
A. serum alpha fetoprotein levels
B. serum human chorionic gonadotropin hormone
C. CT scan of the pelvis
D. ultrasound of the testicles
Explanations
(u) A. Although tumor markers are useful to follow patients with testicular cancer, they are not used as the initial
screen.
(u) B. See A for explanation.
(u) C. CT scanning of the pelvis is most commonly used in the evaluation for metastatic disease and not used in the
initial screen for testicular cancer.
(c) D. An ultrasound of the testes will enable the clinician to discriminate between testicular tumors and epididymitis,
orchitis, hematomas, hydroceles, and infiltrative diseases of the testes.
Ref: (1)

62. Diagnosis/ENT/Ophthalmology
A patient is evaluated in the office with a red eye. The patient awoke with redness and a watery discharge from the
eye. The eyelids were not matted together. Examination reveals a palpable preauricular node. Which of the following
is the most likely diagnosis?
A. bacterial conjunctivitis
B. viral conjunctivitis
C. allergic conjunctivitis
D. gonococcal conjunctivitis

27

Explanations
(u) A. Bacterial conjunctivitis is associated with purulent, not watery eye discharge.
(c) B. Viral conjunctivitis is associated with copious watery discharge and preauricular adenopathy.
(u) C. Allergic conjunctivitis is associated with symptoms limited to the conjunctiva with hyperemia and edema.
(u) D. Gonococcal conjunctivitis is associated with copious purulent discharge and no preauricular adenopatthy.
Ref: (28)

63. Clinical Therapeutics/Urology/Renal


Which of the following treatments of constipation should be used with extreme caution in patients who have chronic
renal insufficiency?
A. milk of magnesium
B. psyllium (Metamucil)
C. docusate sodium (Colace)
D. lactulose (Chronulac)
Explanations
(c) A. Patients with chronic renal insufficiency have difficulty excreting magnesium and hypermagnesemia almost
always occurs in a patient with chronic renal insufficiency.
(u) B. Psyllium is a fiber rich product that helps to bulk up the stool. It can be safely used in the constipated renal
patient.
(u) C. Docusate sodium is a stool softener that does not accumulate in the patient with renal insufficiency so it can be
safely used.
(u) D. Lactulose is an agent that is an unabsorbable sugar that can be safely used to treat constipation in the patient
with chronic renal insufficiency.
Ref: (1)

64. Diagnostic Studies/Neurology


A 2 month-old infant has had a single, generalized tonic-clonic convulsion lasting 4 to 5 minutes. There is no history
of trauma and the infant had been well previously. Physical findings include a temperature of 39.6 degrees C (103.2
degrees F), a bulging tympanic membrane on the right, and an inflamed pharynx. The next most appropriate step is
to
A. perform a lumbar puncture.
B. order x-ray studies of the skull.
C. obtain an electroencephalogram.
D. send home with antibiotics and an anticonvulsant.
Explanations
(c) A. Febrile convulsions are uncommon under the age of 3 months. The physical findings suggest the possibility of
meningitis, so lumbar puncture with CSF analysis is indicated.
(u) B. See A for explanation.
(u) C. See A for explanation.
(h) D. See A for explanation. In addition, anticonvulsant therapy is not indicated for febrile seizures.
Ref: (13)

65. Scientific Concepts/Gastrointestinal/Nutritional


Which of the following subtypes of viral hepatitis requires the presence of the hepatitis B virus for replication?
A. hepatitis A
B. hepatitis C
C. hepatitis D
D. hepatitis E
Explanations
(u) A. Hepatitis A is transmitted almost exclusively by the fecal-oral route and does not require the presence of the
hepatitis B virus for replication.
(u) B. While hepatitis C may occur in IV drug users, it does not require the presence of the hepatitis B virus for
replication.

28

(c) C. In the United States, hepatitis D is most commonly seen in persons exposed frequently to blood or blood
products, such as drug addicts. It requires the presence of hepatitis B virus for its replication and expression.
(u) D. Hepatitis E resembles hepatitis A in its enteric mode of transmission and does not require the presence of the
hepatitis B virus for replication.
Ref: (11)

66. Clinical Intervention/Orthopedics/Rheumatology


A 34 year-old female, with a past medical history of irritable bowel syndrome and migraines, presents with fatigue,
generalized aching and stiffness of the trunk, hip, and shoulder girdles. She complains of pain and tightness in the
neck and across the upper posterior shoulders. She complains of poor sleep, but denies depression. Physical
examination is unremarkable except for numerous tender points on palpation. Laboratory evaluation was
unremarkable. Which of the following is the best intervention for this patient?
A. corticosteroids
B. supervised exercise program
C. hydrocodone (Vicodin)
D. refer to endocrinologist
Explanations
(u) A. Corticosteroids have no role in the management of fibromyalgia.
(c) B. A carefully planned and individualized exercise program has been proven effective for the management of
fibromyalgia.
(u) C. Opioids are not first-line agents in the treatment of fibromyalgia.
(u) D. One-third of patients with fibromyalgia are found to have deficiency of growth hormone, but referral to
endocrinology is not indicated at this time.
Ref: (11)

67. Diagnosis/Pulmonology
A 60 year-old patient returned from the recovery room to the floor following a subtotal gastrectomy. At 3 AM the next
morning, the patient's temperature is 102 F (39 C) and pulse is 112/min. Which of the following is the most likely
cause?
A. wound infection
B. atelectasis
C. phlebitis
D. shock
Explanations
(u) A. Wound infection does not present this early.
(c) B. Atelectasis is the most common pulmonary complication, affecting 25% of patients with abdominal surgery. It is
more common in elderly and overweight patients and occurs within the first 12 to 24 hours postoperatively.
(u) C. Phlebitis occurs more commonly after the second postoperative day.
(u) D. In shock, the pulse is usually thready and the temperature is not elevated.
Ref: (31)

68. Diagnosis/Pulmonology
A patient who appears very anxious enters the office complaining of dizziness with perioral and extremity
paresthesias. She vaguely describes some chest discomfort. Physical examination is unremarkable, except for
moderate tachypnea with obvious sighing respiration. This clinical picture is most consistent with
A. bronchial asthma.
B. hyperventilation syndrome.
C. spontaneous pneumothorax.
D. emphysema.
Explanations
(u) A. Bronchial asthma attacks are associated with increased dyspnea and prolonged expiration. Patients may use
accessory muscles of respiration as part of this acute condition.

29

(c) B. Anxiety may result in hyperventilation that can result in perioral numbness and paresthesias of the extremities.
These paresthesias are due to decreased CO2 in the blood stream that results from the hyperventilation. Anxious
patients also will have nondescript chest pain as part of this condition and may also complain of dizziness.
(u) C. Spontaneous pneumothorax patients will primarily complain of significant chest pain along with their dyspnea.
These patients will not have perioral or extremity paresthesias.
(u) D. Emphysema alone will not result in hyperventilation or the production of perioral or extremity paresthesias and
is a chronic progressive rather than an acute onset condition.
Ref: (7)

69. Scientific Concepts/Cardiology


A patient presents with moderate mitral stenosis. Which of the following complications is associated with an
increased risk of systemic embolization in this patient?
A. atrial fibrillation
B. pulmonary hypertension
C. increased left atrial pressure
D. left ventricular dilatation
Explanations
(c) A. 50-80% of patients with mitral stenosis will develop paroxysmal or chronic atrial fibrillation; 20-30% of patients
with atrial fibrillation will have systemic embolization.
(u) B. Pulmonary hypertension can occur in patients with severe mitral stenosis with symptoms of low cardiac output
and right sided heart failure. Pulmonary hypertension does not cause systemic embolization.
(u) C. Patients with mitral stenosis can have increased left atrial pressures relative to the left ventricular diastolic
pressures; this does not usually cause systemic embolization.
(u) D. Left ventricular dilatation is more common in aortic valve disease than mitral valve disease.
Ref: (28)

70. Diagnosis/Neurology
A 37 year-old male presents with daytime fatigue and drowsiness. He states that he does not sleep well, with
frequent awakenings during the night. He has gained 8 pounds over the past six months, and he complains of
palpitations. His wife states that he snores at night. A home nocturnal pulse oximetry indicates that his saturation
drops 6% intermittently throughout the night. Which of the following is the most likely diagnosis?
A. depression
B. narcolepsy
C. hypothyroidism
D. obstructive sleep apnea
Explanations
(u) A. Hypothyroidism, narcolepsy, and depression are not associated with oxyhemoglobin desaturation or snoring.
(u) B. See A for Explanation.
(u) C. See A for Explanation.
(c) D. Obstructive sleep apnea is associated with obesity, nighttime wakening and snoring, hypertension, cardiac
dysrhythmias, and oxyhemoglobin desaturation of greater than 4% during hypopnea or apnea.
Ref: (11)

71. Clinical Therapeutics/Gastrointestinal/Nutritional


A 62 year-old patient with arthritis is on daily naproxen therapy. Which of following medications is used to protect
against GI complications?
A. diphenoxylate (Lomotil)
B. sulfasalazine (Azulfidine)
C. metoclopramide (Reglan)
D. misoprostol (Cytotec)
Explanations
(u) A. Diphenoxylate is used in the treatment of diarrhea.
(u) B. Sulfasalazine is used in the treatment of inflammatory bowel disease.

30

(u) C. Metaclopramide is used in the treatment of nausea and vomiting.


(c) D. Misoprostol is an analog of prostaglandins and used in the treatment of NSAID induced ulcer disease.
Ref: (15)

72. Diagnostic Studies/Cardiology


A 19 year-old female presents with complaint of palpitations. On examination you note the patient to have particularly
long arms and fingers and a pectus excavatum. She has a history of joint dislocation and a recent ophthalmologic
examination revealed ectopic lentis. Which of the following echocardiogram findings would be most consistent with
this patient's physical features?
A. right atrial enlargement
B. aortic root dilation
C. pulmonic stenosis
D. ventricular septal defect
Explanations
(u) A. Patients with Marfan's syndrome commonly have mitral valve prolapse and possibly aortic regurgitation. Right
atrial enlargement, pulmonic stenosis and ventricular septal defect are not commonly seen.
(c) B. This patient has the signs and symptoms consistent with Marfan's syndrome. Ectopia lentis, aortic root dilation
and aortic dissection are major criteria for the diagnosis of the disease.
(u) C. See A for explanation.
(u) D. See A for explanation.
Ref: (28)

73. Diagnosis/Orthopedics/Rheumatology
A 14 year-old male who is overweight presents with complaints of left knee and anteromedial thigh pain for the past
month. He states the pain gets better with rest and denies any known trauma. On examination of the gait, a slight
limp is noted. X-ray films of the left knee are normal. The most likely diagnosis is
A. genu valgum.
B. Legg-Calve-Perthes disease.
C. Osgood-Schlatter disease.
D. slipped capital femoral epiphysis.
Explanations
(u) A. Genu valgum is a knock-knee deformity of the knees and would be detected on physical examination.
(u) B. Legg-Calve-Perthes presents in a younger population.
(u) C. Osgood-Schlatter disease is characterized by local pain, swelling, and tenderness to palpation overlying the
tibial tubercle, and x-ray findings of tibial tubercle prominence, with or without free bony fragments.
(c) D. Slipped capital femoral epiphysis is most common in overweight adolescent males who present with complaints
of pain that is referred to the thigh or medial side of the knee associated with a limp. X-ray films of the knee are
normal since the condition involves the hip.
Ref: (5)

74. Diagnostic Studies/Psychiatry/Behavioral Medicine


A patient with advanced AIDS complicated by toxoplasmosis presents with altered mental status, recent onset of
seizures, and focal neurologic deficits. Which of the following diagnostic studies is most helpful?
A. EEG
B. MRI
C. lumbar puncture
D. Toxoplasma gondii antibody titers
Explanations
(u) A. An EEG is useful to diagnose epilepsy, but the MRI will demonstrate the cause of the seizures in this case.

31

(c) B. An MRI showing multiple isodense or hypodense ring-enhancing mass lesions is the most useful test for such a
patient.
(h) C. Lumbar puncture is contraindicated secondary to possible mass effect.
(u) D. Antibody titers cannot be depended upon since most patients have IgG titers that reflect past infection,
significant rises are infrequent, and IgM antibody is rare.
Ref: (11)

75. Diagnosis/Infectious Diseases


A patient presents with a rash six days after returning from a camping and hiking trip in the woods. The rash is
macular and first appeared on the ankles and then the rest of the lower extremities. On exam the physician assistant
notes lesions on the soles of the feet and the trunk. Which of the following is the most likely diagnosis?
A. Rocky mountain spotted fever
B. typhoid fever
C. Lyme disease
D. Q fever
Explanations
(c) A. Rocky mountain spotted fever presents with a macular rash on the wrists, ankles, extremities, and trunk. After 5
days the rash appears on the palms and soles.
(u) B. The rash of typhoid fever is a faint, salmon-colored, maculopapular rash. The rash is noted primarily on the
trunk and chest. The palms and soles are spared.
(u) C. Lyme disease presents with a red macule or papule rash which expands slowly with central clearing at the site
of the tick bite. Palms and soles are spared.
(u) D. Cattle, sheep, and goats transmit Q fever. The rash is non-specific and not seen on the palms or soles.
Ref: (7)

76. Health Maintenance/Pulmonology


Which of the following is the most effective way for patients with persistent asthma to monitor the severity of their
symptoms?
A. call the health care provider regularly
B. keep a diary of symptoms
C. monitor peak flow
D. ask a family member to monitor symptoms
Explanations
(u) A. Calling the health care provider regularly or asking a family member to monitor symptoms is not effective in
patients understanding how to manage their asthma symptoms.
(u) B. Keeping a diary may be effective for patients to understand their cause of symptoms, but would not be effective
in helping to manage their symptoms.
(c) C. Monitoring peak flow is the most effective way for the patients and health care providers to manage symptoms
and guide treatment.
(u) D. See A for explanation.
Ref: (7)

77. Clinical Therapeutics/ENT/Ophthalmology


A patient with Type 1 diabetes mellitus was treated for otitis externa of the right ear for 2 weeks with topical ear
drops. The patient presents today with persistent, foul aural discharge, granulations in the ear canal, and deep ear
pain. Which of the following is the proper treatment at this time?
A. ciprofloxacin (Cipro) IV
B. cefuroxime (Zinacef) IV
C. ampicillin-sulbactam (Unasyn) PO
D. azithromycin (Zithromax) PO
Explanations
(c) A. IV antibiotics directed against Pseudomonas, the most likely etiology, is needed for the treatment of malignant
otitis media.

32

(u) B. Cefuroxime, ampicillin-sulbactam, and azithromycin have no activity against Pseudomonas.


(u) C. See B for explanation.
(u) D. See B for explanation.
Ref: (11)

78. Clinical Therapeutics/Endocrinology


A 52 year-old male, who underwent transphenoidal surgery and radiation for acromegaly 15 years ago, presents with
the following exam findings: weight gain of 20 lbs, puffy eyes, coarse dry skin and normal visual fields. Laboratory
findings reveal the following: TSH 0.3 u/ml (normal 0.5-5.0), free T4 0.3 ng/ml (normal 0.7-2.7). Which of the
following drugs would be indicated for this patient?
A. levothyroxine (Synthroid)
B. growth hormone
C. PTU (Propylthiouracil)
D. bromocriptine (Parlodel)
Explanations
(c) A. The patient is suffering from hypothyroidism as indicated by the low free T4. TSH is not elevated in this case
secondary to the previous history of transphenoidal surgery and radiation for his acromegaly.
(u) B. See A for explanation.
(u) C. PTU is utilized for hyperthyroidism.
(u) D. Bromocriptine is utilized for treatment of prolactinoma.
Ref: (28)

79. Diagnosis/Pulmonology
A 79 year-old female presents with productive cough for 2 days. She has associated fever, chills and shortness of
breath. On physical exam, RR 30, BP 90/60, T 101.3. There is no JVD. Lungs reveal crackles at the left lower lobe
and decreased breath sounds with dullness to percussion. Heart exam reveals RRR with no S3 or S4. No edema is
noted. On chest x-ray the patient has a left sided pleural effusion. Examination of the pleural fluid reveals a
decreased glucose and an elevated pleural fluid LDH. Pleural fluid cytology reveals squamous epithelial cells. What is
the most likely cause of the patient's effusion?
A. malignancy
B. bacterial pneumonia
C. heart failure
D. pulmonary embolus
Explanations
(u) A. Malignancy is also a leading cause of an exudative pleural effusion, second to bacterial pneumonia. Cytology in
this case was normal.
(c) B. Bacterial pneumonia is the leading cause of an exudative pleural effusion.
(u) C. Pleural effusions in heart failure are transudative, not exudative pleural effusions.
(u) D. PE can be both an exudative and transudative classification of effusion, however, dyspnea is usually the most
common symptom.
Ref: (7)

80. Diagnosis/Cardiology
A patient presents with chest pain. ECG done in the emergency department reveals ST segment elevation in leads II,
III, and AVF. This is most consistent with a myocardial infarction in which of the following areas?
A. anterior wall
B. inferior wall
C. posterior wall
D. lateral wall
Explanations
(u) A. Anterior wall myocardial infarction is characterized by ST segment elevation in 1 or more of the precordial (V1V6) leads.
(c) B. Inferior wall myocardial infarction is characterized by ST segment elevation in leads II, III, and AVF.

33

(u) C. Posterior wall myocardial infarction is characterized by ST segment depression in leads V1-V3 and a large R
wave in leads V1-V3.
(u) D. Lateral wall myocardial infarction is characterized by ST segment elevation in leads I and AVL.
Ref: (7)

81. Scientific Concepts/Obstetrics/Gynecology


Which of the following is the most significant factor in the production of menopausal symptoms?
A. decrease in estrogen
B. decrease in testosterone
C. increase in progesterone
D. increase in androstenedione
Explanations
(c) A. A decrease in estrogen is responsible for the majority of menopausal symptoms including, but not limited to, hot
flashes, sleep disturbances, genital tract atrophy and mood changes.
(u) B. While there is a slight decrease in testosterone levels during menopause, this has not be shown to be
responsible for the majority of menopausal symptoms.
(u) C. Progesterone and androstenedione levels both decrease, not increase, in the menopausal female patient and
therfore can not account for any menopausal symptoms.
(u) D. See C for explanation.
Ref: (4)

82. Clinical Therapeutics/Neurology


A 53 year-old woman recently diagnosed with Parkinson's disease has a moderate tremor. She has no other signs of
disease at this time. Which of the following is the most appropriate initial treatment?
A. lorazepam (Ativan)
B. haloperidol (Haldol)
C. ramatidine (Flumadine)
D. levodopa/carbidopa (Sinemet)
Explanations
(u) A. Benzodiazepines are used to treat anxiety and insomnia, but are not used to treat Parkinson's disease.
(u) B. Haloperidol is used for the treatment of Tourette's syndrome, but is not indicated for the treatment of
Parkinson's disease.
(u) C. Ramatidine is used in the treatment of viral infections and has no role in the treatment of Parkinson's disease.
(c) D. Levodopa/carbidopa is first-line therapy for the treatment of Parkinson's disease.
Ref: (28)

83. Diagnostic Studies/Orthopedics/Rheumatology


A 23 year-old female presents with ongoing arthralgias with intermittent flares of arthritis. She is found to have a
malar rash and an abnormal urinalysis. Serum ANA and anti-double-stranded DNA antibodies are present. Which of
the following tests should be ordered to assess her risk for thrombotic events and future risk of spontaneous
abortion?
A. complete blood count
B. PT/INR
C. bleeding time
D. anti-phospholipid antibodies
Explanations
(u) A. CBC may reveal anemia, leukopenia and thrombocytopenia, but these are not the cause of thrombotic events
and spontaneous abortion in SLE.
(u) B. The PT/INR should not be altered in SLE.
(u) C. Bleeding time measures platelet function, not risk for thrombosis.
(c) D. Anti-phospholipid antibodies are present in 25% of SLE patients and may cause thrombotic events and
spontaneous abortion.
Ref: (11)

34

84. History & Physical/Endocrinology


Which of the following are characteristics of Cushing's syndrome?
A. central obesity, glucose intolerance, and easy bruising
B. attacks of severe headaches, hypertension, and glucosuria
C. hyperpigmentation, hypoglycemia, and orthostasis
D. tetany, hypocalcemia, and cataracts
Explanations
(c) A. These are classic signs of Cushing's syndrome.
(u) B. These are suggestive of pheochromocytoma.
(u) C. These are suggestive of Addison's disease.
(u) D. These are suggestive of hypoparathyroidism.
Ref: (28)

85. Clinical Therapeutics/Cardiology


Which of the following is an absolute contraindication to thrombolytic therapy in a patient with an acute ST segment
elevation myocardial infarction?
A. history of severe hypertension presently controlled
B. current use of anticoagulation therapy
C. previous hemorrhagic stroke
D. active peptic ulcer disease
Explanations
(u) A. See C for explanation.
(u) B. See C for explanation.
(c) C. Absolute contraindications to thrombolytic therapy include a previous hemorrhagic stroke, a stroke within one
year, a known intracranial neoplasm, active internal bleeding, and a suspected aortic dissection. Severe, but
controlled hypertension, use of anticoagulation, and active peptic ulcer disease are relative contraindications in which
the risk/benefit ratio must be weighed in each patient.
(u) D. See C for explanation.
Ref: (28)

86. Scientific Concepts/Orthopedics/Rheumatology


Which of the following fractures is associated with the greatest risk of avascular necrosis of the femoral head?
A. intertrochanteric
B. femoral neck
C. subtrochanteric
D. pelvic rim
Explanations
(u) A. See B for explanation.
(c) B. Femoral neck fractures lead to the greatest disruption of arterial blood supply to the femoral head.
(u) C. See B for explanation.
(u) D. See B for explanation.
Ref: (26)

87. Scientific Concepts/ENT/Ophthalmology


Bitemporal hemianopia is noted on physical examination in a patient with visual changes over the past 2 years. The
central field of vision is spared. The lesion is located in the
A. optic nerve.
B. optic chiasm.
C. temporal optic radiation.
D. optic tract.

35

Explanations
(u) A. A lesion in the optic nerve would result in loss of vision in the affected eye only and include loss of central
vision.
(c) B. A lesion in the optic chiasm would result in the loss of vision in the bilateral temporal fields and spare the
central field of vision.
(u) C. A lesion in the temporal optic radiation would produce superior contralateral quadrantopia.
(u) D. A lesion in the optic tract would result in loss of vision in the temporal field of the ipsilateral eye.
Ref: (23)

88. History & Physical/Gastrointestinal/Nutritional


The initial manifestation of acute pancreatitis is often
A. generalized pruritus.
B. epigastric pain.
C. epigastric mass.
D. anorexia.
Explanations
(u) A. Pruritus presents as a symptom only when acute pancreatitis occurs as a result of obstruction of the distal
common bile duct.
(c) B. The typical findings in acute pancreatitis include nausea, vomiting, and abdominal pain. Epigastric pain,
generally abrupt in onset, is steady and severe.
(u) C. Epigastric mass is more common in pancreatic cancer.
(u) D. Anorexia is a common finding with chronic pancreatitis, but uncommon with acute pancreatitis.
Ref: (11)

89 History & Physical/Orthopedics/Rheumatology


A 22 year-old male presents to the ED after sustaining a blow to the knee during football practice. The knee exam
demonstrates significant forward translation of the tibia when the knee is in 20 degrees of flexion and the tibia is
forced forward while the femur is stabilized. Which of the following knee maneuvers does this represent?
A. abduction stress test
B. anterior drawer sign
C. Lachman test
D. McMurray test
Explanations
(u) A. The abduction stress test is performed to evaluate medial collateral ligament tears while applying valgus stress.
(u) B. The anterior drawer sign is performed to evaluate the anterior cruciate ligament; however the patient is supine,
hips and knees flexed, and feet are flat on the table.
(c) C. The Lachman test is performed to evaluate the anterior cruciate ligament. The knee is placed in 20 degrees of
flexion.
(u) D. The McMurray test is performed to evaluate medial and lateral meniscal tears while rotating the lower leg
internally and externally.
Ref: (3)

90. History & Physical/Orthopedics/Rheumatology


Which of the following would demonstrate rotational misalignment in a patient with a fracture of the fourth
metacarpal?
A. base of the ring fingernail and index fingernail line up in the partially closed hand
B. fingernails of the open hand form an asymmetric arc
C. ring finger of the closed hand overlaps the little finger
D. ring finger of the open hand is shortened

36

Explanations
(u) A. Failure of the planes of the fingernails in the partially closed hand to line up indicates rotational misalignment.
(u) B. Fingernails of the open hand normally form an asymmetric arc.
(c) C. All fingernails should point to the same spot when the hand is closed. Overlapping of one finger over the other
indicates rotational misalignment.
(u) D. Shortening of a finger does not indicate rotational misalignment.
Ref: (17)

91. Clinical Therapeutics/Urology/Renal


A 34 year-old male presents with symptoms of painful urethral discharge. History reveals recent (7 days ago)
intercourse with a new partner. A Gram stain is negative for intracellular diplococci. Assuming no allergies, which of
the following is the antibiotic of choice?
A. doxycycline
B. penicillin G
C. ciprofloxacin (Cipro)
D. cephalexin (Keflex)
Explanations
(c) A. Tetracyclines, such as doxycycline, are drugs of first choice for Chlamydia.
(u) B. Penicillin G is not used in treating chlamydial infections.
(u) C. Cephalexin and ciprofloxacin are not effective in treating Chlamydial infections.
(u) D. See C for explanation.
Ref: (28)

92. Health Maintenance/Psychiatry/Behavioral Medicine


A 28 year-old male patient is being treated for depression and has been taking paroxetine (Paxil) for the past two and
a half months with a marked improvement in symptoms. The patient reports problems with sexual functioning which
he believes is related to the medication. Which of the following is an immediate concern with abrupt discontinuation of
the medication?
A. increased risk of suicide
B. diminished sexual functioning
C. risk of drug withdrawal symptoms
D. worsened depressive symptoms
Explanations
(u) A. Abrupt discontinuation of an SSRI in a patient who is relatively asymptomatic does not increase the risk of
suicide.
(u) B. Abrupt discontinuation may improve sexual functioning after the medication has cleared.
(c) C. The greatest risk of abrupt withdrawal of short acting SSRIs is a withdrawal syndrome or withdrawal delirium
which represents cholinergic rebound.
(u) D. Approximately 50% of patients with a first episode of depression are at risk for relapse at some time during
their lifetime; however, relapse is not an immediate risk of abrupt discontinuation.
Ref: (14)

93. Clinical Intervention/Urology/Renal


A 16 year-old male presents with increasing pain and swelling of his right scrotum. The right testicle is extremely
tender to palpation on examination. A Doppler ultrasound demonstrates decreased blood flow. Which of the following
is the most appropriate intervention?
A. oral doxycycline
B. emergent surgery
C. incision and drainage
D. scrotal elevation and ice packs
Explanations
(h) A. Oral doxycycline is the treatment of choice for epididymitis. While epididymitis and testicular torsion present
similarly, the Doppler ultrasound in epididymitis would show increased blood flow, not decreased.

37

(c) B. Once a diagnosis of testicular torsion is suspected, emergent surgery is indicated to have the best possible
chance of salvaging the testicle (85-97% chance if less than 6 hours). Any other treatment measures delay the
definitive treatment and increase the risk of testicular ischemia and infarction.
(h) C. Incision and drainage is indicated for treatment of abscesses, not testicular torsion.
(h) D. Scrotal elevation and ice packs are indicated for adjunct treatment of epididymitis, not testicular torsion.
Ref: (28)

94. Health Maintenance/Cardiology


A postmenopausal woman is at greatest risk of death from which of the following?
A. stroke
B. heart disease
C. ovarian cancer
D. breast cancer
Explanations
(u) A. See B for explanation.
(c) B. Although women tend to be concerned about dying from breast cancer, heart disease is the number one killer
of postmenopausal women.
(u) C. See B for explanation.
(u) D. See B for explanation.
Ref: (11)

95. Health Maintenance/Urology/Renal


The most effective preventive strategy to prevent recurrence of renal lithiasis is which of the following?
A. increase in hydration
B. early treatment of urinary tract infection
C. limitation of calcium intake
D. use of probenecid
Explanations
(c) A. Keeping the urine dilute is the most effective strategy to prevent crystal accumulation in the urine and the
development of urinary stones.
(u) B. Treating UTI only affects the formation of calcium pyrophosphate or struvite stones.
(u) C. Limitation of calcium in the diet reduces the ability of calcium to bind oxalate leading to the production of
calcium oxalate stones.
(u) D. Probenecid is a uricosuric agent that promoted uric acid secretion in the urine that may promote more uric acid
stones.
Ref: (28)

96. Diagnosis/Cardiology
A 46 year-old female is being evaluated for a new-onset hypertension that was discovered on screening at her
workplace. The patient had several readings revealing systolic and diastolic hypertension. Patient is currently on no
medications. Physical examination is unremarkable. A complete laboratory evaluation revealed hypokalemia as the
only abnormality. Which of the following is the most likely diagnosis for this patient?
A. pheochromocytoma
B. renal artery stenosis
C. coarctation of the aorta
D. primary aldosteronism
Explanations
(u) A. Pheochromocytoma will result in an increase in the production and release of catecholamines, which results in
an increase in urinary metanephrines on testing.
(u) B. Renal artery stenosis is identified by an abnormal radionuclide uptake on the affected kidney.
(u) C. Coarctation of the aorta is identified by delayed and weakened femoral pulses along with a blood pressure in
the lower extremities significantly lower than in the upper extremities.

38

(c) D. Primary aldosteronism has an increased aldosterone secretion, which causes the retention of sodium and the
loss of potassium. This should be the primary consideration for this patient.
Ref: (11)

97. History & Physical/Obstetrics/Gynecology


Which of the following is associated with meconium-stained amniotic fluid during labor?
A. transition
B. prematurity
C. fast labor
D. fetal distress
Explanations
(u) A. Transition is the last phase of the first stage of labor. Complete cervical dilation marks the end of this stage of
labor.
(u) B. Meconium passage occurs most commonly in post-term deliveries, not pre-term deliveries. Passage of
meconium is related to mature development of the gastrointestinal tract and is rarely seen before 36 weeks gestation.
(u) C. Prolonged labor, not fast labor, is associated with potential passage of meconium into the amniotic fluid.
(c) D. Passage of meconium is associated with fetal distress usually due to asphyxia.
Ref: (8)

98. Diagnosis/ENT/Ophthalmology
A 23 year-old graduate student presents with sudden onset of severe dizziness, with nausea and vomiting for the
past couple of hours. She denies hearing loss or tinnitus. She has had a recent cold. Which of the following is the
most likely diagnosis?
A. Meniere's disease
B. vestibular neuronitis
C. benign positional vertigo
D. vertebrobasilar insufficiency
Explanations
(u) A. Meniere's disease is associated with hearing loss, tinnitus, and vertigo that lasts from seconds to hours.
(c) B. Vestibular neuronitis or labyrinthitis presents with vertigo, nausea, and vomiting, but not hearing loss or tinnitus.
It is related to viral URIs, and develops over several hours, with symptoms worse in the first day, with gradual
recovery over several days.
(u) C. Benign positional vertigo occurs with changes in position, especially rapid movements of the head. Nausea
may occur, but vomiting is not significant.
(u) D. Vertebrobasilar insufficiency is usually accompanied by brain stem findings, such as diplopia, dysarthria, or
dysphagia, and is not common in this age group.
Ref: (11)

99. Clinical Intervention/Endocrinology


A diabetic patient returns for follow-up of non-fasting blood work done at a local health fair. The total cholesterol is
230 mg/dl. Which of the following is the appropriate next step?
A. referral to a cardiologist
B. reassurance
C. obtain a fasting HDL and LDL lipid measurement
D. start a HMG Co-A reductase inhibitor
Explanations
(u) A. See C for explanation.
(u) B. See C for explanation.
(c) C. A patient with elevated total cholesterol needs further evaluation through a fasting total lipid profile including
LDL and HDL.
(u) D. See C for explanation.
Ref: (28)

39

100. Clinical Intervention/Orthopedics/Rheumatology


A 32 year-old medical transcriptionist presents with burning and tingling in her right wrist and hand for the past month.
On physical exam, Phalen's test is positive; however, there is no atrophy of the thenar eminence. Which of the
following is the initial step in management of this patient?
A. wrist splints
B. corticosteroid injection
C. surgical referral
D. propoxyphene (Darvocet)
Explanations
(c) A. The treatment of carpal tunnel syndrome is aimed at relieving the pressure on the median nerve. This is best
accomplished by having the patient wear a wrist splint during the activities that increase the pressure on the median
nerve.
(u) B. Corticosteroid injections and surgery are indicated only after a trial of the wrist splint provides no relief.
(u) C. See B for explanation.
(u) D. Darvocet has no role in the treatment of carpal tunnel syndrome.
Ref: (28)

101. Diagnostic Studies/Urology/Renal


A urinalysis performed during a routine physical examination on a 43 year-old male reveals 1-2 hyaline casts/HPF.
The remainder of the UA is normal. Based upon these results, the physician assistant should
A. collect a urine for culture and sensitivity.
B. do nothing, since these casts are considered normal.
C. refer the patient to a nephrologist.
D. schedule the patient for a CT scan.
Explanations
(u) A. See B for explanation.
(c) B. Hyaline casts are not indicative of renal disease. They can be found following strenuous exercise and with
concentrated urine or during a febrile illness.
(u) C. See B for explanation.
(u) D. See B for explanation.
Ref: (24)

102. Diagnostic Studies/Orthopedics/Rheumatology


An x-ray reveals a break in the cortex of one side of the ulna shaft without a separation or break of the opposite
cortex describes what type of fracture?
A. greenstick
B. transverse
C. torus (buckle)
D. epiphyseal
Explanations
(c) A. A greenstick fracture is a break in the cortex of one side of bone shaft without a break in the opposite cortex.
(u) B. A transverse fracture is a complete fracture of both cortices.
(u) C. A torus fracture is a bowing, bending, or buckling without a break in the cortex.
(u) D. Epiphyseal fracture occurs at the growth plate.
Ref: (29)

103. Clinical Intervention/Cardiology


A 54 year-old female who has diabetes presents with rubor, absence of hair, and brittle nails of her left foot. She
complains of leg pain that awakens her at night. Examination reveals a femoral bruit with diminished popliteal and
pedal pulses on the left side. The most appropriate therapy would be

40

A. vasodilator therapy.
B. bypass surgery.
C. exercise program.
D. embolectomy.
Explanations
(u) A. Vasodilator therapy is not indicated.
(c) B. Bypass surgery is indicated in the presence of rest pain and provides relief of symptoms in 80 to 90% of
patients.
(u) C. While an exercise program is appropriate with claudication, rest pain is a surgical indication.
(u) D. Embolectomy is used for acute arterial occlusion.
Ref: (28)

104. Clinical Therapeutics/Dermatology


What scabicide has been associated with neurotoxicity in infants and young children?
A. lindane (Kwell)
B. crotamiton (Eurax)
C. 10% sulfur ointment
D. permethrin (Elimite)
Explanations
(c) A. Lindane (Kwell) is concentrated in the CNS and toxicity from systemic absorption in infants has been reported.
(u) B. Crotamiton (Eurax) is not associated with CNS toxicity and is an effective scabicide. Its primary side effects
include dermatitis and conjunctivitis.
(u) C. Sulfur ointment is no longer used because newer agents have been developed and is not neurotoxic.
(u) D. Permethrin (Elimite) is the drug of choice for the treatment of scabies and is not associated with neurotoxicity.
Ref: (10)

105. Clinical Therapeutics/Cardiology


Which electrolyte abnormality is associated with an increase in the risk for digoxin toxicity?]
A. hypercalcemia
B. hypokalemia
C. hypermagnesemia
D. hyponatremia
Explanations
(u) A. See B for explanation.
(c) B. Decreased concentration of potassium results in the increased activity of cardiac glycosides by increasing
tissue binding and decreasing renal excretion of digoxin. Potassium loss is the only significant electrolyte abnormality
that significantly affects digoxin metabolism.
(u) C. See B for explanation.
(u) D. See B for explanation.
Ref: (15)

106. History & Physical/Pulmonology


Kussmaul breathing is characterized by
A. rapid, deep labored breathing.
B. irregular and varying depth of breathing.
C. frequently interspersed deeper breaths.
D. periods of deep breathing alternate with periods of apnea.
Explanations
(c) A. Kussmaul breathing is characterized by rapid deep labored breathing.
(u) B. This characterizes ataxic breathing.

41

(u) C. This characterizes sighing.


(u) D. Periods of deep breathing alternate with periods of apnea is Cheyne-Stokes breathing.
Ref: (3)

107. Clinical Therapeutics/ENT/Ophthalmology


Which of the following may precipitate acute angle-closure glaucoma?
A. metoclopramide
B. timolol
C. glyburide
D. acetazolamide
Explanations
(c) A. Metoclopramide and other drugs with high anticholinergic effects may precipitate acute angle-closure glaucoma
from pupillary dilation.
(u) B. Timolol, a beta-antagonist, is used in the treatment of acute angle-closure glaucoma.
(u) C. Glyburide has no relationship to glaucoma.
(u) D. Acetazolamide, a carbonic anhydrase inhibitor, may suppress the production of aqueous humor by 40-60% and
is used in the emergency treatment of glaucoma.
Ref: (23)

108. Clinical Intervention/Dermatology


A mother brings in her 3 month-old infant and states that she has noticed a rash on her infant's scalp. Physical
examination reveals the presence of erythematous and scaling crusty lesions involving the vertex of the scalp. Which
of the following is the most appropriate initial intervention?
A. selenium sulfide shampoo
B. permethrin 1% cream rinse
C. warm olive oil compresses
D. scrubbing of scalp with hexachlorophene
Explanations
(u) A. While selenium sulfide shampoos can be used in treatment of scalp seborrheic dermatitis in adults, it is not
recommended for use in infants and young children.
(u) B. Permethrin 1% cream rinse is utilized in the treatment of pediculosis, not seborrheic dermatitis.
(c) C. This infant most likely has scalp seborrheic dermatitis ("cradle cap"). Initial treatment consists of warm olive oil
compresses to remove any crusts followed by use of baby shampoo or mild hydrocortisone cream.
(u) D. Hexachlorophene is a bacteriostatic skin cleanser which is not indicated in the treatment of seborrheic
dermatitis. It is also not recommended for use in infants and young children.
Ref: (10)

109. Clinical Intervention/Pulmonology


A 75 year-old man with a long history of COPD presents with acute onset of worsening dyspnea, increased
productive cough, and marked agitation. While in the emergency department he becomes lethargic and obtunded.
His ABG's reveal a PaO2 40 mmHg, PaCO2 65 mmHg, and arterial pH 7.25. Which of the following is the most
appropriate management at this point?
A. oxygen supplementation with a 100% non-rebreather mask
B. noninvasive positive pressure ventilation (NIPPV)
C. endotracheal intubation and mechanical ventilation
D. emergency tracheostomy
Explanations
(u) A. Supplemental oxygen and positive pressure ventilation are inadequate for patients with overt respiratory failure.
(u) B. See A for explanation.
(c) C. This patient is in severe respiratory arrest with markedly impaired mental status; conventional mechanical
ventilation is required.
(h) D. Tracheostomy is indicated for an obstructed airway.
Ref: (7)

42

110. Clinical Intervention/Psychiatry/Behavioral Medicine


Which of the following is the most appropriate intervention in suspected child abuse?
A. Arrange for the arrest of the parents.
B. Confront the suspected assailant in front of the child.
C. Assure the safety of the child, with hospitalization if necessary.
D. Contact social service department after discharge of the child.
Explanations
(u) A. Law enforcement would be contacted after an investigation of the incident(s) by the proper authorities. It is not
the responsibility of the medical caregiver to arrange for the arrest, only to report your suspicions.
(u) B. Confrontation should be done by the investigators of the incident(s).
(c) C. The primary goal should be the safety and well-being of the child. Hospitalization may be the only way the
clinician has to remove the child from the care of a possible abusive home if no other recourse is available due to a
lack of social services and investigators.
(u) D. Any abuse suspicion should be reported prior to the discharge of the child. Discharging the child to the person
or people responsible for the abuse may cause greater harm to the child.
Ref: (5)

111. Health Maintenance/Cardiology


A 56 year-old male, status post myocardial infarction, is noted to have left ventricular hypertrophy and an ejection
fraction of 38%. Which of the following medications should be prescribed to prevent the development of heart failure
symptoms?
A. amlodipine (Norvasc)
B. furosemide (Lasix)
C. hydrochlorothiazide (HCTZ)
D. lisinopril (Zestril)
Explanations
(u) A. See D for explanation.
(u) B. See D for explanation.
(u) C. See D for explanation.
(c) D. ACE inhibitors have been shown to markedly improve survival and are also recommended for prevention of
symptoms in patients at risk for heart failure.
Ref: (28)

112. History & Physical/ENT/Ophthalmology


A patient presents with the complaint of irritation of the left eye one day after gardening. He states "I think there is
something in my eye." Which of the following findings is consistent with your suspected diagnosis?
A. increased intraocular pressure
B. rust ring
C. hazy cornea
D. fluorescein uptake
Explanations
(u) A. Elevated intraocular pressure is seen with glaucoma.
(u) B. Rust ring is seen with metallic foreign bodies.
(u) C. Hazy cornea is seen with glaucoma.
(c) D. Fluorescein dye uptake is diagnostic for corneal abrasion.
Ref: (29)

113. Scientific Concepts/Infectious Diseases


The causative organism for molluscum contagiosum is which of the following?
A. virus
B. gram-negative bacterium
C. gram-positive bacterium
D. parasite

43

Explanations
(c) A. Molluscum contagiosum is caused by a poxvirus.
(u) B. See A for explanation.
(u) C. See A for explanation.
(u) D. See A for explanation.
Ref: (28)

114. Diagnostic Studies/Dermatology


A 13 year-old child presents with a lesion on his right forearm that is occasionally pruritic. On examination, a 2 cm
ring of erythema with a scaly border and central clearing is noted. A suspected diagnosis is confirmed by the
presence of
A. a positive patch test.
B. hyphae on a KOH prep.
C. gram-positive cocci on Gram stain.
D. multinucleated cells on Tzanck smear.
Explanations
(u) A. Patch testing is positive with allergic dermatitis, not fungal disorders.
(c) B. Ring-shaped lesions with scaly borders and central clearing are most likely caused by fungal infection.
Microscopic examination of scrapings reveals hyphae on KOH prep.
(u) C. A positive Gram stain is found with bacterial skin infections, not fungal.
(u) D. Multinucleated cells found on Tzanck smear indicate herpes, not fungal infections.
Ref: (10)

115. Scientific Concepts/Urology/Renal


Which of the following is the most common composition of kidney stones?
A. calcium oxalate
B. uric acid
C. struvite
D. calcium phosphate
Explanations
(c) A. Approximately three fourths of all kidney stones are comprised of calcium oxalate.
(u) B. See A for explanation.
(u) C. See A for explanation.
(u) D. See A for explanation.
Ref: (1)

116. Clinical Therapeutics/Orthopedics/Rheumatology


A 67 year-old female presents with progressive pain in her left knee that is worse with activity and relieved with rest.
She notes stiffness of the knee that last about 20 minutes after activity is resumed. She exercises regularly. No
known drug allergies. On physical exam she is 5' 5", 225 pounds. Her left knee exam reveals mild effusion without
erythema or warmth. Radiographs of the left knee reveal medial joint space narrowing and subchondral bone
sclerosis. Her sodium is 138 mEq/L, potassium 4.3 mEq/L, bicarbonate 24 mEq/L, chloride 104 mEq/L, BUN 23 mg/dl
and creatinine 1.8 mg/dl. Which of the following medications is most appropriate for this patient's worsening pain?
A. naprosyn
B. prednisone
C. acetaminophen
D. methotrexate
Explanations
(u) A. Naprosyn is contraindicated in patients with abnormal renal function.
(u) B. Oral prednisone is not appropriate for the long-term management of osteoarthritis.
(c) C. Acetaminophen is recommended as first-line pharmacotherapy in patients with osteoarthritis.
(u) D. Methotrexate is not indicated for the treatment of osteoarthritis.
Ref: (11)

44

117. Diagnosis/Orthopedics/Rheumatology
An 18 year-old patient has a tibia/fibula fracture following a motorcycle crash. Twelve hours later the patient presents
with increased pain despite adequate doses of analgesics and immobilization. Which of the following is the most
likely diagnosis?
A. avascular necrosis
B. myositis ossificans
C. compartment syndrome
D. reflex sympathetic dystrophy
Explanations
(u) A. Avascular necrosis is a late complication of fracture resulting from disruption of the blood supply to the bone.
(u) B. Myositis ossificans occurs primarily in muscles post-traumatically and may not arise for several months after an
injury.
(c) C. Compartment syndrome is characterized by a pathological increase of pressure within a closed space and
results from edema or bleeding within the compartment. It may occur as an early local complication of fracture.
(u) D. Reflex sympathetic dystrophy is characterized by painful wasting of muscles that may be secondary to injury
and could occur as a late complication.
Ref: (26)

118. Diagnosis/Endocrinology
A woman brings her 3 month-old son to the clinic. Upon examination, it is noted he has a round face, a large
protruding tongue, dry skin, an umbilical hernia, and his weight gain is below average. He appears apathetic and the
mother says the infant is usually constipated. Which of the following is the most likely diagnosis?
A. hyperparathyroidism
B. nephrotic syndrome
C. phenylketonuria
D. hypothyroidism
Explanations
(u) A. Hyperparathyroidism results in abnormal bone development, nausea, vomiting, and anorexia.
(u) B. Nephrotic syndrome is associated with proteinuria, with resultant edema and ascites. Anorexia, abdominal pain
and diarrhea are common findings.
(u) C. Phenylketonuria is associated with mental retardation, motor deficits, and convulsions.
(c) D. Congential hypothyroidism presents gradually, and at 3-6 months findings include poor appetite and feeding,
sluggishness, constipation, enlarged abdomen and umbilical hernia, enlarged tongue, and the child does not meet
developmental milestones.
Ref: (5)

119. Health Maintenance/Cardiology


A 74 year-old patient presents with signs and symptoms of heart failure. EKG shows the patient to be in atrial
fibrillation at a rate of 80 bpm. Blood pressure is 120/76. The patient denies complaint of palpitations, chest pain, or
syncope. Which of the following is the most important long term therapy in this patient?
A. verapamil (Calan)
B. amiodarone (Cordarone)
C. furosemide (Lasix)
D. warfarin (Coumadin)
Explanations
(u) A. Calcium channel blockers are utilized in rate control of atrial fibrillation. This patient's rate is controlled at
80bpm presently.
(u) B. Antiarrhythmic therapy may be considered in patients with atrial fibrillation; however anticoagulation therapy
must occur first.
(u) C. Diuretics may be indicated in the acute treatment of heart failure; however they may not be needed long term.
(c) D. Patients with atrial fibrillation have an increased risk for stroke, therefore these patients need anticoagulation
with warfarin to an INR of 2.0-3.0.
Ref: (28)

45

120. Diagnostic Studies/Obstetrics/Gynecology


A 29 year-old female presents for routine prenatal visit at 26 weeks gestation. She has no complaints and has
completed all the initial routine obstetrical diagnostic tests to date. Her physical examination and all initial diagnostic
evaluations are unremarkable. Which of the following is the most appropriate diagnostic test to order at this time?
A. VDRL
B. amniocentesis
C. maternal serum alpha-fetoprotein
D. 1-hour post-Glucola blood glucose
Explanations
(u) A. A serological test for syphilis, usually the VDRL, is part of the routine obstetrical tests ordered at a patient's
initial prenatal visit.
(u) B. Genetic testing should be offered routinely to patients over the age of 35. Amniocentesis is usually performed
routinely at 16-18 weeks gestation if indicated.
(u) C. Maternal serum alpha-fetoprotein testing is routinely done between 15-18 weeks gestation to screen for neural
tube defects.
(c) D. Glucose screening, usually with a 1-hour Glucola, is routinely performed between 24-28 weeks gestation to
evaluate for glucose intolerance.
Ref: (4)

121. Clinical Therapeutics/Gastrointestinal/Nutritional


Which of the following is the mechanism of action of infliximab (Remicade)?
A. block lymphocyte proliferation
B. direct effect on smooth muscle
C. monoclonal antibody that binds to tumor necrosis factor
D. 5-HT antagonist
Explanations
(u) A. The immunomodulators (6-MP) work by blocking lymphocyte proliferation.
(u) B. The mechanism of action of mebeverine is direct effect on smooth muscle in irritable bowel syndrome.
(c) C. Infliximab is a monoclonal antibody that binds to tumor necrosis factor.
(u) D. The anti-nausea and vomiting medications like ondansetron work by blocking 5-HT receptors.
Ref: (11)

122. Diagnostic Studies/Cardiology


Which of the following ECG findings is consistent with hyperkalemia?
A. prolonged QT interval
B. delta wave
C. peaked T waves
D. prominent U waves
Explanations
(u) A. Prolonged QT interval is seen in hypocalcemia.
(u) B. Delta wave is a sign of ventricular preexcitation seen in Wolf-Parkinson-White (WPW) Syndrome.
(c) C. Narrowing and peaking of T waves are the beginning EKG changes associated with hyperkalemia.
(u) D. Prominent U waves are a sign of prolonged ventricular repolarization seen in hypokalemia.
Ref: (28)

123. Clinical Therapeutics/Hematology


A 35 year-old female presents with fatigue. CBC results reveal the following:
WBC: 6,300/microliter
Hgb: 9.5 g/dl
Hct: 28%
MCV: 75 fL
MCHC: 32 g/dl
MCH: 24 pg
Platelets: 550,000/mL

46

Which of the following is the best treatment option for this patient?
A. folic acid
B. vitamin B12
C. prednisone
D. ferrous sulfate
Explanations
(u) A. Vitamin B12 and folate deficiency present with macrocytic cells and are treated with vitamin B12 and folate
respectively.
(u) B. See A for explanation.
(u) C. Prednisone is used to treat immune-mediated hemolytic anemias which present with normocytic,
normochromic red blood cells.
(c) D. Iron deficiency anemia is a microcytic, hypochromic anemia and is treated with ferrous sulfate.
Ref: (28)

124. History & Physical/Infectious Diseases


Which of the following is typically noted on physical examination in a patient with diphtheria?
A. papular rash on trunk
B. supraclavicular adenopathy
C. pharyngeal pseudomembranes
D. splenomegaly
Explanations
(u) A. See C for explanation.
(u) B. See C for explanation.
(c) C. The classic exam finding noted in diphtheria is a gray pharyngeal pseudomembrane. Rash, splenomegaly, and
supraclavicular adenopathy are not noted in diphtheria.
(u) D. See C for explanation.
Ref: (7)

125 Health Maintenance/Obstetrics/Gynecology


Which of the following is a major risk factor for an ectopic pregnancy?
A. alcohol intake
B. young maternal age
C. history of salpingitis
D. low dose oral contraceptive use
Explanations
(u) A. Alcohol intake has not been associated with an increased risk of ectopic pregnancy.
(u) B. Advanced maternal age, not younger maternal age, is an established risk factor for ectopic pregnancy.
(c) C. A previous history of salpingitis is a major risk factor for ectopic pregnancy since damage to the fallopian tube
prevents the fertilized ovum from reaching the uterus prior to implantation.
(u) D. Oral contraceptive use prevents ovulation and therefore decreases the over-all risk of pregnancy, including
ectopic pregnancies. While high levels of estrogen and progesterone are thought possibly to increase the risk of
ectopic pregnancy because these hormones slow the movement of the fertilized egg through the fallopian tube, no
proven association has been established.
Ref: (4)

126. Health Maintenance/Pulmonology


Which of the following forms of lung cancer is associated with the poorest prognosis?
A. squamous cell
B. adenocarcinoma
C. large cell
D. small cell

47

Explanations
(u) A. See D for explanation.
(u) B. See D for explanation.
(u) C. See d for explanation.
(c) D. Small cell lung cancer is the most common type of lung cancer that is metastatic at the time of discovery, and
therefore has the poorest prognosis.
Ref: (7)

127. Clinical Intervention/ENT/Ophthalmology


A 28 year-old HIV positive male presents complaining of fever, dysphagia, odynophagia, and trismus for the past 2
days. Physical examination reveals an ill-appearing male with a temperature of 101.3 degrees F, poor dentition,
"woody" edema in the sublingual area and neck, tongue displaced posteriorly, and drooling. The immediate
managment of this patient includes which of the following?
A. CT of the neck
B. intubation
C. start IV penicillin
D. incision and drainage of the abscess
Explanations
(u) A. See B for explanation.
(c) B. This person presents with the classic signs and symptoms of Ludwig's angina. He is unable to handle his
secretions and the displacement of his tongue suggests impending airway obstruction. Intubation to secure his airway
is the most immediate concern. CT of the neck for diagnosis, starting IV antibiotics, and incision and drainage are
actions that need to be taken after securing the airway.
(u) C. See B for explanation.
(u) D. See B for explanation.
Ref: (7)

128. Diagnosis/Endocrinology
A 32 year-old female presents to the ED unconscious. Blood glucose obtained from a finger stick reveals the glucose
to be 28 mg/dl. Further blood work obtained at IV line placement confirms the glucose finding. Immediate
resuscitation is given with D50 and the patient responds appropriately. History taken from the patient is significant for
episodes of feeling faint and weak over the past 2-4 weeks, she denies however any medical problems, or the use of
any medications. Additional laboratory results from the blood taken in the ED reveal a serum insulin level to be 55
u/ml (normal less than 22), C-peptide 5.4 ng/ml (normal 0.5-2.0), and proinsulin 1.0 ng/ml (normal 0-0.2). Given
these results what is the most likely diagnosis?
A. insulinoma
B. exogenous insulin administration
C. sulfonylurea usage
D. glucagonoma

Explanations
(c) A. The laboratory results confirm the diagnosis of an insulinoma by having an increase in the insulin level,
increased C-peptide and proinsulin levels.
(u) B. Exogenous insulin administration would be confirmed by the following laboratory results: Insulin levels are
increased; C-peptide and proinsulin are decreased. C-peptide comes from the cleavage of proinsulin to insulin.
Exogenous administration of insulin by-passes this process.
(u) C. Sulfonylurea usage would be confirmed in the history, however the laboratory results would show an increase
in the insulin level and C-peptide level but a normal level of the proinsulin.
(u) D. Glucagonoma is extremely rare and in addition would present with symptomatology of diabetes mellitus by
stimulating glycogenolysis and gluconeogenesis.
Ref: (28)

48

129. Clinical Therapeutics/Pulmonology


A 47 year-old HIV positive female presents with a complaint of a nonproductive cough. She is febrile, tachypneic and
tachycardic. Lung exam reveals bilateral rales. Chest x-ray shows diffuse interstitial infiltrates. What is the
recommended treatment in this patient?
A. trimethoprim-Sulfamethoxazole (Bactrim)
B. tetracycline (Sumycin)
C. amantadine (Symmetrel)
D. ticarcillin (Ticar)
Explanations
(c) A. TMP-SMX is the drug of choice for all forms of pneumocystis.
(u) B. Tetracycline, amantadine and ticarcillin are not effective against PCP.
(u) C. See B for explanation.
(u) D. See B for explanation.
Ref: (7)

130. History & Physical/Cardiology


A 58 year-old male presents with chest pain. Vital signs include blood pressure of 210/175, pulse 80, RR 20. Which
of the following would you expect to find on physical examination?
A. papilledema
B. carotid bruit
C. diastolic murmur
D. absent peripheral pulses
Explanations
(c) A. Malignant hypertension is characterized by marked blood pressure elevation with papilledema, often with
encephalopathy or nephropathy.
(u) B. Carotid bruits are associated with carotid artery stenosis.
(u) C. Diastolic murmurs are associated with valvular heart disease such as aortic regurgitation and mitral stenosis.
(u) D. Peripheral pulses are absent in acute arterial occlusion or severe peripheral arterial disease.
Ref: (28)

131. Clinical Therapeutics/Obstetrics/Gynecology


A 24 year-old female presents with complaints of dysuria associated with fever, malaise, myalgias and headache for
the past 3 days. Today she noticed some lesions on her genitalia. Physical examination reveals the presence of
multiple clear vesicles and erythematous ulcers with tender bilateral inguinal adenopathy. Which of the following is
the most appropriate treatment for this patient?
A. valacyclovir (Valtrex)
B. metronidazole (Flagyl)
C. ceftriaxone (Rocephin)
D. benzathine penicillin G (Bicillin)
Explanations
(c) A. This patient most likely has herpes genitalis which may be treated with oral antivirals, such as valacyclovir, that
will reduce the duration of viral shedding and shorten the duration of symptoms.
(u) B. Metronidazole is the treatment of choice for Trichomonas vaginitis.
(u) C. Ceftriaxone is one of the options to treat uncomplicated gonorrhea or is used as part of the treatment regimen
for pelvic inflammatory disease.
(u) D. Benzathine penicillin G is the treatment of choice for a patient with syphilis.
Ref: (4)

132. Clinical Therapeutics/Neurology


Cognitive loss in Alzheimer's dementia may be delayed with which of the following medications?
A. donepezil (Aricept)
B. haloperidol (Haldol)

49

C. risperidone (Risperdal)
D. zolpidem (Ambien)
Explanations
(c) A. Donepezil is a reversible cholinesterase inhibitor that leads to increased acetylcholine, which is necessary for
learning and memory.
(u) B. Haloperidol, risperidone, and other antipsychotics may be used to treat the agitation and behavioral symptoms
in patients with dementia, but have significant side effects.
(u) C. See B for explanation.
(u) D. Zolpidem is used to treat insomnia.
Ref: (11)

133. Diagnosis/Orthopedics/Rheumatology
A 58 year-old male presents complaining of anterior right shoulder pain the day after performing extensive yard work.
The pain is localized over the anterior proximal humerus with distinct point tenderness. There are no visible
abnormalities. The patient has full range of motion and strength with all shoulder movements. The pain is reproduced
by asking the patient to resist the examiner during supination of the right elbow. Which of the following is the most
likely diagnosis?
A. supraspinatus tendonitis
B. subacromial bursitis
C. rotator cuff tear
D. bicipital tendonitis
Explanations
(u) A. Supraspinatus tendonitis, subacromial bursitis and rotator cuff tear usually present with pain in the area of the
deltoid muscle, limited abduction and are reproduced through impingement tests that narrow the space between the
acromium and the humerus thereby impinging the supraspinatus tendon.
(u) B. See A for explanation.
(u) C. See A for explanation.
(c) D. Bicipital tendonitis presents with anterior shoulder pain that is reproduced by palpating the tendon in the
humeral bicipital groove and through resisted motion of the biceps muscle (elbow flexion or supination).
Ref: (11)

134.
Which of the following findings is consistent with thyrotoxicosis?
A. bradycardia
B. menorrhagia
C. nervousness
D. constipation
Explanations
(u) A. Bradycardia, dry skin, constipation, and menorrhagia are typically associated with hypothyroidism.
(u) B. See A for explanation.
(c) C. Thyrotoxicosis presents with heat intolerance, sweating, nervousness, diarrhea, tachycardia, moist skin, and
tremor.
(u) D. See A for explanation.
Ref: (28)

135. Clinical Therapeutics/Cardiology


A 55 year-old diabetic female presents for a 3 month blood pressure follow-up. At the last visit the BP was 160/90 for
the third consecutive visit. She was placed on an ACE inhibitor and educated regarding lifestyle modifications. At
today's visit the patient complains of persistent annoying dry cough that has been going on since the last visit. BP
today is 120/70. What is the best recommendation to control her BP?
A. add a diuretic
B. stop the ACE inhibitor and continue lifestyle modifications
C. switch patient to an Angiotensin II Receptor Blocker (ARB)
D. do nothing and recheck BP in 3 months

50

Explanations
(u) A. This patient's blood pressure is controlled; there is no indication at this time to add an additional drug.
(u) B. This patient's chronic dry cough is likely secondary to the ACE inhibitor, the medication should be stopped,
however the patient needs something for blood pressure control.
(c) C. This patient's chronic dry cough is likely secondary to the ACE inhibitor, the medication should be stopped.
Angiotensin II Receptor Blockers (ARBs) are similar to ACE inhibitors for BP control, but do not cause cough.
(u) D. This patient's chronic dry cough is likely secondary to the ACE inhibitor, the medication should be stopped to
encourage compliance.
Ref: (28)

136. Diagnosis/Cardiology
A newborn is seen for an initial two week visit. Physical examination reveals a thrill and a continuous machinery
murmur in the left second intercostal space. Which of the following is the most likely diagnosis?
A. patent ductus arteriosus
B. ventricular septal defect
C. tetralogy of Fallot
D. coarctation of the aorta
Explanations
(c) A. Patent ductus arteriosus is characterized by a classic harsh, machinery-like murmur that is continuous through
systole and diastole. This is heard best at the left second interspace and is commonly associated with a thrill.
(u) B. Ventricular septal defect is characterized by a holosystolic murmur at the lower left sternal border.
(u) C. Tetralogy of Fallot is characterized by a systolic thrill at the left sternal border with a systolic ejection murmur
that may or may not have an associated systolic click.
(u) D. Coarctation of the aorta is associated with a systolic ejection click or a short systolic murmur at the left sternal
border.
Ref: (5)

137. Clinical Intervention/ENT/Ophthalmology


A 10 year-old boy was playing with sparklers (magnesium sulfate) and got some of the "sparkle" in his right eye.
Which of the following is the most appropriate initial treatment?
A. irrigate the eye for at least 20 minutes
B. apply Bacitracin ointment and patch the eye
C. remove the sparkle with a moistened cotton swab
D. protect the eye with a metal shield and refer to an eye ophthalmologist
Explanations
(c) A. The magnesium from the sparkler combines with tears, producing an alkaline injury and should be treated with
prolonged irrigation. Irrigation should be the first step in management of this case.
(u) B. See A for explanation.
(u) C. See A for explanation.
(u) D. See A for explanation.
Ref: (23)

138. History & Physical/Gastrointestinal/Nutritional


Ulcerative colitis usually presents with which of the following?
A. bloody diarrhea
B. toxic megacolon
C. fever and left quadrant pain
D. alternating constipation and diarrhea
Explanations
(c) A. Ulcerative colitis typically presents with episodic bloody diarrhea, lower abdominal cramps, and urgency to
defecate.
(u) B. Toxic megacolon is a complication of ulcerative colitis, but it is not a common presentation.
(u) C. Left lower quadrant pain and a palpable mass accompanied by fever is classic for diverticulitis.
(u) D. Irritable bowel syndrome usually presents with constipation, painless diarrhea with mucous, or alternating
constipation and diarrhea.
Ref: (11)

51

139. Scientific Concepts/Dermatology


Which of the following is the most common type of skin cancer?
A. basal cell
B. melanoma
C. atypical nevi
D. squamous cell
Explanations
(c) A. Basal cell cancer is the most common cause of skin cancer usually occurring on sun-exposed areas.
(u) B. While malignant melanoma is the leading cause of death from skin disease, it is not the most common skin
cancer.
(u) C. Atypical nevi are associated with melanoma. They are diagnosed clinically, not histologically. Any atypical nevi
suspected to be melanomas should be removed.
(u) D. Squamous cell carcinomas also occurs in sun-exposed areas, but are less frequent than basal cell cancers.
Ref: (10)

140. History & Physical/ENT/Ophthalmology


Which of the following findings is most consistent with cataracts?
A. conjunctival injection
B. poorly visualized optic disc
C. central visual field loss
D. arcus senilis
Explanations
(u) A. Conjunctival injection is associated with conjunctivitis and other inflammatory conditions of the eye.
(c) B. Cataracts are caused by opacification of the crystalline lens, and this decreases the amount of light that enters
the eye. It is difficult to see through the lens from either direction, and thus, the optic disc is poorly visualized on
examination.
(u) C. Central field loss is associated with macular degeneration.
(u) D. While arcus senilis may be seen in geriatric patients and is not associated with cataracts.
Ref: (23)

141. Clinical Therapeutics/Pulmonology


A 36 year-old male developed a sore throat and was treated with IM penicillin. Within 20 minutes, he felt faint and
became dyspneic. Upon entry to the emergency department, he was pale and apprehensive. He had a thready pulse,
and systolic blood pressure was 40 mmHg. Which of the following is the most appropriate initial agent to use?
A. dopamine
B. epinephrine
C. hydrocortisone
D. diphenhydramine
Explanations
(u) A. Dopamine is not indicated in the treatment of allergic reactions.
(c) B. Epinephrine is the drug of first choice for emergency use and should be given as soon as anaphylactic shock is
suspected or diagnosed.
(u) C. Hydrocortisone should be given as an adjunct to epinephrine, but it is not the drug of first choice.
(u) D. Diphenhydramine should be given as an adjunct to treatment, but it is not the drug of first choice.
Ref: (29)

142. Clinical Therapeutics/Psychiatry/Behavioral Medicine


A child has been under treatment for attention-deficit hyperactivity disorder (ADHD). No response has occurred with
behavioral adaptations. Which of the following categories of medication should this patient be given?
A. stimulants
B. anxiolytics
C. antipsychotics
D. antidepressants

52

Explanations
(c) A. Stimulants, such as methylphenidate (Ritalin), are effective in 50 to 80% of children with ADHD.
(u) B. Anxiolytic agents, such as alprazolam (Xanax), are used in treating anxiety disorders.
(u) C. Antipsychotics, such as chlorpromazine (Thorazine), are used in treating psychosis.
(u) D. Antidepressants, such as amitriptyline (Elavil), are used to treat depression.
Ref: (5)

143. Clinical Intervention/Neurology


A 73 year-old female patient was diagnosed 3 years ago with Alzheimer's disease and heart failure. Her 80 year-old
husband can no longer help feed and bathe her or manage her medications. Which of the following support services
is most appropriate for this patient?
A. hospice care
B. senior center
C. adult day care
D. skilled nursing facility
Explanations
(u) A. Hospice care is a program to provide palliative care to individuals who are terminally ill and projected to die
within 6 months.
(u) B. Senior centers are community-based facilities that provide recreational activities and mid-day meals for
functional adults.
(u) C. Adult day health care is designed to meet the needs of functionally impaired adults in a community setting, but
does not usually provide for the higher level of care that is required to feed or bathe a patient.
(c) D. A skilled nursing facility is appropriate for patients requiring assistance with activities of daily living (i.e., feeding
and bathing) and a higher level of safety assurance.
Ref: (28)

144. Health Maintenance/Infectious Diseases


Polysaccharide pneumococcal vaccine should be given to a patient with which of the following diseases?
A. asthma
B. sickle cell disease
C. acute renal failure
D. osteoarthritis
Explanations
(u) A. Pneumococcal vaccination is not currently recommended for patients with asthma, acute renal failure, or
osteoarthritis.
(c) B. Patients with sickle cell disease are functionally asplenic and, therefore, require pneumococcal vaccination to
prevent invasive disease.
(u) C. See A for explanation.
(u) D. See A for explanation.
Ref: (32)

145. Health Maintenance/Gastrointestinal/Nutritional


A 65 year-old patient with adenocarcinoma of the colon in remission presents for follow-up. Which of the following
tumor markers should be monitored?
A. AFP
B. CEA
C. CA 19-9
D. CA-125
Explanations
(u) A. AFP is used to monitor recurrence of hepatocellular carcinoma.
(c) B. CEA is used to monitor recurrence of colon carcinoma.
(u) C. CA 19-9 is used to monitor recurrence of pancreatic carcinoma.
(u) D. CA-125 is used to monitor recurrence of ovarian carcinoma.
Ref: (28)

53

146. Diagnosis/Psychiatry/Behavioral Medicine


A 26 year-old female has a long history of sexual promiscuity and substance abuse. She frequently expresses anger
when she feels abandoned. She also has difficulty in controlling her anger at times. Her past relationships have been
intense and short-lived. She has attempted suicide twice in the past 18 months. This patient exhibits which of the
following disorders?
A. bipolar
B. avoidant personality
C. borderline personality
D. dissociative
Explanations
(u) A. Bipolar disorder is characterized by swings in mood from extreme elation to severe depression. The
characteristics of this patient do not fit the diagnosis of bipolar disorder.
(u) B. In avoidant personality disorders, individuals show an exaggerated sensitivity to rejection and severe shyness.
(c) C. Borderline personality disorder is characterized by instability of interpersonal relationships, marked impulsivity
that is potentially self-damaging, inappropriate, intense anger or control of anger, recurrent suicidal attempts,
gestures or threats, and identity disturbances. All of these are exhibited in this patient.
(u) D. Dissociative disorder is what is commonly termed "multiple personality disorder." For this diagnosis, the person
must display the presence of two or more distinct identities. These personalities recurrently take control of the
patient's behavior, there is an inability to recall important personal information, and the disorder is not the effect of a
substance or a medical condition.
Ref: (14)

147. History & Physical/Cardiology


A patient had an acute inferior, transmural myocardial infarction 4 days ago. A new murmur raises the suspicion of
mitral regurgitation due to papillary muscle rupture. Which of the following murmur descriptions describes this
condition?
A. A grade III/VI diastolic murmur heard best at the apex without radiation.
B. A grade IV/VI systolic ejection murmur heard best at the base with radiation to the left clavicle.
C. A grade II/VI systolic murmur heard best at the apex preceded by a click and without radiation.
D. A grade IV/VI systolic murmur heard best at the apex with radiation to the left axilla.
Explanations
(u) A. This is a classic description of mitral stenosis.
(u) B. This is a classic description for pulmonic stenosis.
(u) C. This is a classic description for mitral valve prolapse.
(c) D. This is a classic description of mitral regurgitation. The papillary muscle rupture is a complication of an acute
inferior transmural myocardial infarction, and results in a failure of the mitral valve leaflets to close. The direction of
regurgitant flow of blood is toward the left axilla.
Ref: (3)
148. Diagnosis/ENT/Ophthalmology
A 4 year-old boy presents with purulent, foul-smelling nasal discharge for three days. He has not had any other
symptoms of respiratory illness, cough, wheeze, or fever. His activity level and appetite has been normal. On exam,
he is afebrile. TM's have normal light reflex, canals are clear. Left nare is clear; there is considerable amount of
purulent exudate from the right nare, and a bright reflection of light is noticed. Oropharynx is without inflammation or
exudate. Neck is supple, without lymphadenopathy. Which of the following is the most likely diagnosis?
A. viral URI
B. acute sinusitis
C. allergic rhinitis
D. nasal foreign body
Explanations
(u) A. Viral URI does not present with foul-smelling nasal discharge.
(u) B. Acute sinusitis may present with purulent nasal discharge, but the observation of a bright light reflection
suggests a foreign body.
(u) C. Allergic rhinitis is seasonal, associated with sneezing and other allergy-related symptoms.
(c) D. Nasal foreign body is suggested by unilateral nasal obstruction or discharge.
Ref: (5)

54

149. Clinical Intervention/Gastrointestinal/Nutritional


The main complication with the use of transjugular intrahepatic portosystemic shunt (TIPS) procedure is which of the
following?
A. increased portal pressures resulting in further esophageal varices
B. increased portal pressures resulting in a worsening of cirrhosis
C. Budd-Chiari syndrome
D. increased risk of encephalopathy
Explanations
(u) A. TIPS procedures are performed in order to shunt blood away from the liver parenchyma which in essence
lowers portal pressures lessening the risk for esophageal varices.
(u) B. TIPS procedures, since they cause of bypass of the liver parenchyma, result in a lessening of the blood flow
into the liver which does not cause cirrhosis to progress.
(u) C. Budd-Chiari syndrome is a thrombosis of the hepatic vein. It is not a common complication of the TIPS
procedure.
(c) D. TIPS procedures involve the placement of a stent in the liver in order to shunt blood away from the portal vein
into the hepatic vein which bypasses the cirrhotic liver parenchyma. Its main complication is encephalopathy from the
accumulation of toxic substances in the brain since the liver no longer acts as a filter.
Ref: (7)

150. History & Physical/Orthopedics/Rheumatology


A 12 year-old female presents for a routine sports physical. The physical exam reveals asymmetry of the posterior
chest wall on forward bending. This is most consistent with which of the following?
A. spondylolysis
B. spondylolisthesis
C. scoliosis
D. herniated disc
Explanations
(u) A. Spondylolysis presents with limitation of lumbar flexibility and tight hamstring muscles.
(u) B. Spondylolisthesis presents with reduced lumbar lordosis and sacral kyphosis.
(c) C. Asymmetry of the posterior chest wall on forward bending is the most striking and consistent abnormality in
patients with idiopathic scoliosis.
(u) D. Herniated disc presents with lumbar muscle spasm and a positive straight leg test.
Ref: (5)

151. Diagnostic Studies/Endocrinology


Acute adrenal insufficiency is characterized by which of the following laboratory abnormalities?
A. hyperglycemia and glucosuria
B. hyponatremia and hyperkalemia
C. hypercortisolism and hypokalemia
D. neutrophilia with increased bands
Explanations
(u) A. Hypoglycemia is frequently seen in adrenal crisis, while hyperglycemia and glucosuria may suggest Cushing's
syndrome.
(c) B. Hyponatremia and hyperkalemia characterize acute adrenal insufficiency.
(u) C. Hypercortisolism and hypokalemia are common findings of Cushing's syndrome.
(u) D. Eosinophilia and lymphocytosis are characteristic of adrenal crisis, not neutrophilia and increased bands.
Ref: (28)

152. Diagnostic Studies/Orthopedics/Rheumatology


The most reliable site from which to identify the causative organism in cases of osteomyelitis is the
A. base of ulcer.
B. blood.

55

C. sinus tract.
D. bone.
Explanations
(u) A. See C for explanation.
(u) B. While blood cultures are indicated in acute cases of osteomyelitis, they are only positive in 25-50% of pediatric
hematogenous osteomyelitis and 10% of other forms of bone infection.
(u) C. Taking specimens for culture from a sinus tract or the base of an ulcer correlate poorly with organisms infecting
the bone.
(c) D. Samples from needle aspiration of pus in the bone, or from a bone biopsy, are essential to determine the exact
causative agent.
Ref: (11)

153. Clinical Intervention/Cardiology


A 58 year-old male who is otherwise healthy presents with chest pain and is found to have left main coronary artery
stenosis of 75%. The most important aspect of his management now is
A. daily aspirin to prevent MI.
B. nitrate therapy for the angina.
C. aggressive risk factor reduction.
D. referral for coronary artery revascularization.
Explanations
(u) A. See D for explanation.
(u) B. See D for explanation.
(u) C. See D for explanation.
(c) D. Although medical therapy is important, revascularization is indicated when stenosis of the left main coronary
artery is greater than 50%.
Ref: (11)

154. Clinical Therapeutics/Urology/Renal


When treating a patient with BPH who complains of increased frequency, urgency, decreased force of stream and
hesitancy with an alpha-adrenergic blocking agent, the patient should be warned of what possible side effect?
A. hypertensive crisis
B. postural hypotension
C. development of prostate cancer
D. development of testicular cancer
Explanations
(u) A. Alpha-adrenergic agents do not cause hypertension.
(c) B. This class of medications can cause postural hypotension as a result of the decrease in peripheral vascular
resistance and lower arterial blood pressure by causing relaxation of the arterial and venous smooth muscle.
(u) C. Alpha-adrenergic agents have no effect on testosterone levels, which have been suggested as a possible link
to the development of prostate cancer.
(u) D. Alpha-adrenergic agents have no correlation to the development of testicular cancer.
Ref: (28)

155. Clinical Therapeutics/Gastrointestinal/Nutritional


A 42 year-old patient who is being treated for colon cancer with chemotherapy develops nausea and vomiting. Which
of the following drugs would be the most effective in controlling the nausea and vomiting?
A. scopolamine (Scopace)
B. meclizine (Antivert)
C. ondansetron (Zofran)
D. loperamide (Imodium)

56

Explanations
(u) A. Scopolamine and meclizine are effective against motion sickness, but ineffective against substances that act
directly on the chemoreceptor trigger zone.
(u) B. See A for explanation.
(c) C. Ondansetron selectively blocks 5-HT3 receptors in the periphery (visceral afferent fibers) and in the brain
(chemoreceptor trigger zone). It is indicated for use in chemotherapy induced nausea and vomiting.
(u) D. Loperamide has low anti-emetic potency.
Ref: (15)

156. History & Physical/Neurology


Which of the following is the most common finding of multiple sclerosis?
A. facial palsy
B. hearing loss
C. seizures
D. diplopia
Explanations
(u) A. Facial palsy, hearing loss, and seizures are rare with multiple sclerosis.
(u) B. See A for explanation.
(u) C. See A for explanation.
(c) D. Diplopia due to ophthalmoplegia is the most common presenting complaint in patients with multiple sclerosis.
Ref: (28)

157. Clinical Therapeutics/ENT/Ophthalmology


A 23 year-old sexually active female presents with a 4 day history of painless bilateral eye exudates which she
describes as copious. Visual acuity is 20/20, generalized conjunctival inflammation with sparing of the cornea is noted
on physical examination. Gram stain of the exudate reveals gram negative diplococci. Appropriate management of
this case is
A. ceftriaxone (Rocephin).
B. polymyxin ophthalmic drops (Aerosporin).
C. ciprofloxacin (Cipro).
D. doxycycline (Doryx).
Explanations
(c) A. With sparing of the cornea, as in this case, a single 1 gram IM dose of ceftriaxone is sufficient treatment for
ophthalmic gonorrhea. If the cornea is involved, 5 days of IM ceftriaxone would be required.
(u) B. Polymixin is ineffective against gonococcus.
(u) C. Oral ciprofloxacin is not used in cases of gonococcal conjunctivitis.
(u) D. Doxycycline is ineffective against gonococcus.
Ref: (23)

158. Clinical Intervention/Dermatology


A female patient presents with a few common warts on her hands. She has tried over-the-counter preparations with
little success and desires something that might work quicker, but has the least chance of scarring. Which of the
following is the most appropriate intervention to accomplish this goal?
A. cryosurgery
B. dermabrasion
C. electrosurgery
D. topical acyclovir
Explanations
(c) A. Cryosurgery with liquid nitrogen is effective treatment for common warts and has less of a chance of scarring
than electrosurgery.
(u) B. Dermabrasion is a technique primarily utilized in the treatment of pitted acne scars and does not play a role in
treatment of common warts.

57

(u) C. Electrosurgery is more effective in treatment of common warts, but has a greater chance of scarring than
cryosurgery.
(u) D. Topical acyclovir may be indicated in treatment of herpes simplex viral infections, but is not used to treat
common warts caused by human papillomaviruses.
Ref: (10)

159. History & Physical/Pulmonology


A patient presents with respiratory complaints. Chest x-ray reveals calcification of the hilar nodes with an eggshell
pattern. Which of the following occupations is most consistent with these chest x-ray findings?
A. building demolitioners
B. coal miners
C. sandblasters
D. farmers
Explanations
(u) A. Asbestosis is the most likely occupational risk and has an indistinct heart border appearance on CXR described
as looking like "ground glass".
(u) B. The CXR of a coal miner consists of irregular opacities ranging from a reticular pattern to a nodular pattern.
(c) C. Silicosis can occur in sandblasters and produce a CXR appearance of calcification of the hilar nodes with an
"eggshell" pattern with long term exposure
(u) D. Farmers lung results from spores and produces a hypersensitivity pneumonitis. The CXR would show a patchy
fibrosis.
Ref: (7)

160. Scientific Concepts/Psychiatry/Behavioral Medicine


Which of the following effects result from cigarette smoking and contributes to atherogenesis?
A. transient decrease in blood pressure
B. polycythemia with relative hyperoxemia
C. decreased blood viscosity
D. chronic inflammation
Explanations
(u) A. Smoking leads to a transient increase in blood pressure, not a decrease.
(u) B. Smoking induces a hypoxic state, leading to polycythemia. However, because of increased carbon monoxide,
there is still a relative hypoxemia rather than increased levels of oxygen.
(u) C. Smoking leads to increased blood viscosity, not decreased.
(c) D. Cigarette smoking induces a chronic inflammatory state, which is believed to contribute to atherogenesis.
Ref: (11)

161. Diagnostic Studies/Gastrointestinal/Nutritional


A 45 year-old female presents with a ten pound weight loss and recurrent greasy stools mixed with diarrhea. The
patient notes that these symptoms are worse with certain foods. Which of the following laboratory tests should initially
be ordered?
A. anti-endomysial antibodies
B. anti-mitochondrial antibodies
C. anti-glomerular basement membrane antibodies
D. anti-phospholipid antibodies
Explanations
(c) A. Patients with celiac sprue disease are likely to have anti-endomysial antibody formation.
(u) B. Anti-mitochondrial antibodies are present with hypothyroid disease and other autoimmune disease processes.
(u) C. Anti-glomerular basement disease occurs in the kidney and lungs and these antibodies are positive in patients
with Goodpasture's disease.
(u) D. Anti-phospholipid antibodies are seen in patients who have systemic lupus erythematosus.
Ref: (7)

58

162. Health Maintenance/Hematology


A 13 year-old male with sickle cell trait is interested in playing football for his school. Which of the following
considerations should be reviewed with the patient and his parents?
A. avoid dehydration
B. avoid all contact sports
C. begin daily use of aspirin
D. begin hydroxyurea supplements
Explanations
(c) A. No restriction of sports activity is required of patients with sickle cell trait. Hydration is required in all sports
activities.
(u) B. See A for explanation.
(u) C. There is no increased risk of thrombosis so daily use of aspirin is not needed.
(u) D. Hydroxyurea is used in the treatment of sickle cell disease; sickle cell trait requires no treatment.
Ref: (13)

163. Clinical Therapeutics/Infectious Diseases


A 5 year-old boy presents to the office for follow-up of complaints of pruritus in the perianal area. A cellophane tape
test is positive. Which of the following is the treatment of choice?
A. mebendazole (Vermox)
B. metronidazole (Flagyl)
C. chloroquine (Aralen)
D. paromomycin (Humantin)
Explanations
(c) A. Treatment of choice for enterobiasis (pinworms) is mebendazole.
(u) B. Metronidazole can be used in the treatment of protozoal infections but it is not indicated in the treatment of
pinworms.
(u) C. Chloroquine is an amebacide that interferes with parasite protein synthesis. Its main indications are in the
treatment of malaria and amebiasis.
(u) D. Paromomycin is an aminoglycoside that acts directly on amoebas and against normal and pathogenic
organisms in the GI tract.
Ref: (28)

164. Diagnostic Studies/Cardiology


A 17 year-old woman presents to the office with recurrent episodes of palpitations and near syncope. Initial ECG was
normal. She is concerned about these episodes since they can occur at any time. Which of the following is the most
appropriate step to pursue in her evaluation?
A. cardiac catheterization
B. tilt table testing
C. echocardiogram
D. Holter monitoring
Explanations
(u) A. A cardiac catheterization will not be useful since the patient is at low risk for actual coronary artery disease.
(u) B. Tilt table testing is useful only in trying to determine vasodepressor syncope that is related to position.
(u) C. An echocardiogram shows valves and left ventricle function, not pathways of conduction.
(c) D. Holter monitoring will identify the heart rhythm; an event recorder may also be useful in this setting if the Holter
monitor is not diagnostic.
Ref: (11)

165. Scientific Concepts/Pulmonology


Which of the following describes the pathophysiological changes of pulmonary sarcoidosis?
A. granulomas and inflammation of alveoli, small bronchi and small blood vessels
B. inflammation and destruction of the structural components of the bronchial wall
C. increased permeability of the alveolar-capillary membrane and diffuse alveolar damage
D. edema of the airways with eosinophils, neutrophils, and lymphocytes

59

Explanations
(c) A. Sarcoidosis is characterized by granulomas and inflammation of alveoli, small bronchi and small blood vessels.
(u) B. Bronchiectasis is characterized by inflammation and destruction of the structural components of the bronchial
wall.
(u) C. Acute respiratory distress syndrome is characterized by increased permeability of the alveolar-capillary
membrane and diffuse alveolar damage.
(u) D. Asthma is characterized by edema of the airways with eosinophils, neutrophils, and lymphocytes.
Ref: (7)

166. Clinical Therapeutics/Orthopedics/Rheumatology


Which of the following osteoporosis therapies is so poorly absorbed that it must be taken alone and on an empty
stomach?
A. raloxifene (Evista)
B. alendronate (Fosamax)
C. calcitonin (Miacalcin)
D. teriparatide (Forteo)
Explanations
(u) A. Raloxifene, calcitonin, and teriparatide lack significant drug-drug or drug-food interactions.
(c) B. Fosamax should not be coadministered with any other medication or food.
(u) C. See A for explanation.
(u) D. See A for explanation.
Ref: (15)

167. Diagnosis/Hematology
A 26 year-old female required 12 units packed red blood cells during a trauma resuscitation and surgical repair of
liver and splenic lacerations. The patient is now 6 hours postoperative and has blood oozing from the suture line and
IV sites. There is bloody urine in the Foley bag. Laboratory evaluation demonstrates a platelet count of
10,000/microliter, prolonged prothrombin level, and the presence of fibrin split products. Which of the following is the
most likely diagnosis?
A. acute ABO incompatibility reaction
B. disseminated intravascular coagulation
C. exacerbation of idiopathic thrombocytopenia
D. inadequate repair of the liver lacerations
Explanations
(u) A. ABO incompatibility results in immediate hemolysis and shock.
(c) B. Disseminated intravascular coagulation is characterized by bleeding from many sites as all coagulation factors
are consumed and then broken down, leading to decreased fibrinogen level and platelet count, prolonged PT and
PTT, and presence of fibrin split products.
(u) C. Idiopathic thrombocytopenia is characterized by decreased platelet count, but coagulation factors are normal.
(u) D. Bleeding would be localized only and would result in shock if lacerations of the liver were not repaired properly.
Ref: (11)

168. Diagnosis/Obstetrics/Gynecology
A 27 year-old G1P0 female presents complaining of painless spotting since this morning. She is known to be 12
weeks pregnant. Pelvic examination reveals the presence of blood within the vagina with a closed cervical os. The
uterus is consistent with a 10-12 week gestation and nontender to palpation. Which of the following is the most likely
diagnosis?
A. inevitable abortion
B. threatened abortion
C. incomplete abortion
D. complete abortion

60

Explanations
(u) A. Vaginal bleeding and cramp-like lower abdominal pain are usually present in an inevitable abortion. The
cervical os is also frequently partially open.
(c) B. Vaginal bleeding that occurs prior to the 20th week of gestation is classified as a threatened abortion. Pain is
usually not a major feature and vaginal examination usually reveals a closed cervical os.
(u) C. Vaginal bleeding accompanied by cramp-like pain, cervical dilatation, and passage of some products of
conception constitutes an incomplete abortion.
(u) D. After all products of conception are passed, the uterus contracts and vaginal bleeding stops. The cervical os
closes, but the uterus is smaller than the suspected gestational age following a complete abortion.
Ref: (4)

169. Health Maintenance/Cardiology


A 37 year-old female with history of Turner's syndrome and coarctation of the aorta repaired at the age of 3 presents
for routine examination. The patient is without complaints of chest pain, dyspnea, palpitations, or syncope. On
examination vitals signs reveal a BP of 130/76, HR 70, regular, RR 16. On cardiac examination you note a grade II/VI
systolic ejection murmur at the left sternal border and a grade III/VI blowing diastolic murmur. Which of the following
does this patient require?
A. antibiotic prophylaxis
B. beta blocker therapy
C. chest CT
D. exercise stress test
Explanations
(c) A. This patient has a history of congenital heart disease and presently has a murmur consistent with aortic
regurgitation. This patient requires antibiotic prophylaxis against infective endocarditis.
(u) B. Beta blocker therapy may increase the amount of regurgitation because of increased diastolic time and is not
indicated in this patient.
(u) C. This patient should undergo serial echocardiograms, chest CT will not give information regarding any changes
in the aortic regurgitation or ejection fraction.
(u) D. This patient is without any complaints; exercise stress test is not indicated.
Ref: (28)

170. Diagnostic Studies/Obstetrics/Gynecology


A 65 year-old postmenopausal female presents with complaints of new onset of vaginal bleeding. She relates a
history of two episodes of vaginal bleeding during the past month, each lasting about four days. Which of the
following diagnostic evaluations is the most reliable to evaluate the suspected diagnosis?
A. Pap smear
B. endometrial biopsy
C. serum prolactin level
D. serum FSH and LH levels
Explanations
(u) A. Pap smears are the diagnostic tool of choice to evaluate for cervical cancer. The Pap smear is not reliable in
diagnosing endometrial cancer, but in a small percentage of patients may show atypical endometrial cells which
necessitates follow-up with an endometrial biopsy.
(c) B. Postmenopausal bleeding is a primary complaint in patients with endometrial cancer. The only reliable method
of diagnosis is by endometrial biopsy.
(u) C. Serum prolactin levels are utilized in the evaluation of a patient with galactorrhea, amenorrhea or infertility. It is
elevated most commonly in patients with a pituitary adenoma.
(u) D. Serum FSH and LH levels will be increased due to the patient's menopausal state, but will not help in the
evaluation of the postmenopausal bleeding.
Ref: (4)

171. Diagnostic Studies/Gastrointestinal/Nutritional


Which of the following is used to screen for malabsorption disorders of the intestines?
A. serum gastrin level
B. urea breath test
C. fecal eosinophils
D. stool fecal fat

61

Explanations
(u) A. Serum gastric level is elevated in Zollinger-Ellison syndrome.
(u) B. Urea breath test is used in the diagnosis of H. pylori infection.
(u) C. Eosinophils are elevated in inflammatory diarrhea due to eosinophilic gastroenteritis.
(c) D. Stool fecal fat is the gold standard test for the evaluation of patients with malabsorption of the intestine.
Ref: (11)

172. Diagnostic Studies/Cardiology


A 60 year-old male with history of hypertension and hyperlipidemia presents with intermittent chest heaviness for one
month. The patient states he has had occasional heaviness in his chest while walking on his treadmill at home or
shoveling snow. He also admits to mild dyspnea on exertion. His symptoms are relieved with 2-3 minutes of rest. He
denies lightheadedness, syncope, orthopnea or lower extremity edema. Vitals reveal a BP of 130/90, HR 70, regular,
RR 14. Cardiac examination revealed a normal S1 and S2, without murmur or rub. Lungs were clear to auscultation.
Extremities are without edema. EKG reveals no acute change and cardiac enzymes are negative. Which of the
following is the most appropriate next diagnostic study?
A. cardiac catheterization
B. nuclear exercise stress test
C. helical (spiral) CT
D. transthoracic Echocardiogram
Explanations
(u) A. Coronary angiography is indicated in patients with classic stable angina who are severely symptomatic despite
medical therapy and are being considered for percutaneous intervention (PCI), patients with troublesome symptoms
that are difficult to diagnose, angina symptoms in a patient who has survived sudden cardiac death event, patients
with ischemia on noninvasive testings, a stress test is a better initial diagnostic study for this patient.
(c) B. This patient has signs and symptoms of classic angina; nuclear stress testing is the most useful noninvasive
procedure for diagnosis of ischemic heart disease and evaluation of angina in this patient.
(u) C. Helical CT is used in the diagnosis of pulmonary embolism, not in the evaluation of angina.
(u) D. Echocardiogram is used in the evaluation of valvular heart disease not in the evaluation of suspected
myocardial ischemia.
Ref: (28)

173. Diagnosis/Dermatology
A young child is brought to the clinic because the mother noticed a rash while bathing the child. There is a very red
slightly raised eruption on the child's face across both cheeks. The child has been in good health and does not
appear ill today. The most likely diagnosis is
A. scarlet fever.
B. rubella.
C. roseola.
D. erythema infectiosum.
Explanations
(u) A. Scarlet fever is associated with fever, pharyngitis, and "sandpaper rash" over face, neck, trunk, and extremities.
(u) B. Rubella presents with round or oval maculopapular lesions that begin on the neck or face and spread to the
extremities.
(u) C. Roseola presents with a high fever with few symptoms, followed by the appearance of a macular rash over the
trunk and neck.
(c) D. Erythema infectiosum is generally asymptomatic, presenting with red papules on the face that coalesce to give
a "slapped cheek" appearance.
Ref: (10)

174. History & Physical/Psychiatry/Behavioral Medicine


A 19 year-old woman has been consuming up to six beers daily since she was 16. She is now pregnant with her first
child, has had little prenatal care, and is due to deliver in four weeks. Of the following, which neonatal problems
should you anticipate?
A. kernicterus
B. hydrocephalus
C. low birth weight
D. teeth discoloration

62

Explanations
(u) A. Kernicterus is typically due to ABO/Rh incompatibility, not fetal alcohol syndrome.
(u) B. While a wide variety of disorders, such as infection, tumors, and congenital malformations, may play a role in
the development of hydrocephalus, it is not a feature of fetal alcohol syndrome.
(c) C. Fetal alcohol syndrome is a common cause of low birth weight.
(u) D. Teeth discoloration is common in infants exposed to tetracycline.
Ref: (5)

175. Clinical Intervention/Obstetrics/Gynecology


A 32 year-old female G4P4 with a history of multiple sexual partners is seen in follow-up for recent findings of high
grade squamous intraepithelial lesions (HSIL) on a Pap smear. Which of the following is the most appropriate next
step?
A. HPV DNA testing
B. pelvic ultrasound
C. repeat Pap smear in 4-6 months
D. colposcopy with endocervical curettage
Explanations
(u) A. HPV DNA testing is recommended for initial evaluation of an ASC-US result on a Pap smear. If HPV testing is
positive, colposcopy is then indicated. If negative, the Pap smear is repeated in 12 months.
(u) B. A pelvic ultrasound or abdominopelvic CT scan is indicated when repeat abnormalities are seen on a Pap
smear, however all diagnostic studies have failed to reveal the source of cellular abnormality.
(u) C. Repeat Pap smears in 4-6 months may be done in evaluation of ASC-US Pap smear results after treatment
with either appropriate therapy for vaginal infection or intravaginal estrogen therapy for menopausal patients with
atrophic changes.
(c) D. Colposcopy with endocervical curettage and directed biopsy of suspicious lesions is the appropriate evaluation
of a high grade squamous intraepithelial lesions (HSIL) Pap smear result.
Ref: (4)

176. Scientific Concepts/Pulmonology


The most common pathogen identified in community acquired pneumonia (CAP) is
A. Mycoplasma pneumoniae.
B. Staphylococcus aureus.
C. Legionella pneumophila.
D. Streptococcus pneumoniae.
Explanations
(u) A. M. pneumoniae, S. aureus, Legionella pneumophila are common pathogens for CAP, but do not account for a
majority of the cases.
(u) B. See A for explanation.
(u) C. See A for explanation.
(c) D. S. pneumoniae accounts for a majority of cases of community acquired pneumonia.
Ref: (7)

177. History & Physical/Gastrointestinal/Nutritional


A patient presents with abdominal pain. On deep palpation of the left lower quadrant the patient notes increased pain
in the right lower quadrant. This is noted as a positive
A. Psoas sign.
B. Murphy's sign.
C. Rovsing's sign.
D. Obturator sign.
Explanations
(u) A. A psoas sign is positive when the supine patient notes pain with flexion of the hip against resistance.
(u) B. A Murphy's sign is positive when the patient experiences abrupt cessation of deep breathing with palpation of
the gallbladder.

63

(c) C. A Rovsing's sign is positive when the patient experiences right lower quadrant pain with deep palpation of the
left lower quadrant.
(u) D. The obturator sign is positive when the supine patient experiences pain when the right leg is flexed at the hip
and knee and the hip is externally and internally rotated.
Ref: (3)

178. Diagnostic Studies/Endocrinology


Which of the following laboratory assays demonstrates excessive amounts of catecholamine degradation products in
the urine?
A. dexamethasone suppression
B. metanephrine and normetanephrine
C. creatinine clearance
D. Schilling
Explanations
(u) A. Dexamethasone Suppression Test is utilized to assess for suppression of ACTH. This test is often utilized to
assess for conditions of hypersecretion of glucocorticoids from the adrenal gland.
(c) B. Metanephrine and normetanephrine are the degradation products that result from the metabolism of
epinephrine and norepinephrine.
(u) C. Creatinine Clearance is done to assess glomerular filtration rate.
(u) D. The Schilling Test is done to assess for pernicious anemia specifically as it relates to the absorption of Vitamin
B12.
Ref: (28)

179. Health Maintenance/Pulmonology


In patients with COPD, which of the following has been shown to decrease rate of malignancy and cardiovascular
disease and improve survival?
A. bronchodilator therapy
B. pulmonary rehabilitation
C. oral glucocorticosteroids
D. smoking cessation
Explanations
(u) A. Bronchodilator therapy is used for symptomatic treatment in patients with COPD.
(u) B. Pulmonary rehabilitation improves quality of life, dyspnea and exercise capacity. It also has been shown to
reduce the rate of hospitalization.
(u) C. Chronic use of oral glucocorticosteroids is not recommended because of an unfavorable benefit/risk ratio.
(c) D. Smoking cessation has been shown to provide significant improvement in decreasing the rate of decline in
pulmonary function.
Ref: (7)

180. History & Physical/Cardiology


A 28 year-old patient presents with complaint of chest pain for two days. The patient describes the pain as constant
and sharp. It is worse with lying down, better with sitting up and leaning forward. Vital signs are BP 120/80, HR 80,
regular, RR 14 and Temperature 100.1 degrees F. Which of the following would you expect to find on physical
examination?
A. lower extremity edema
B. carotid bruit
C. pericardial friction rub
D. splinter hemorrhages
Explanations
(u) A. Lower extremity edema is seen with heart failure or venous insufficiency, not pericarditis.
(u) B. Carotid bruits are associated with carotid artery stenosis, not pericarditis.
(c) C. This patient has signs and symptoms of pericarditis. A pericardial friction rub is characteristic of acute
pericarditis.
(u) D. Subungual (splinter) hemorrhages are characteristic of infective endocarditis, not pericarditis.
Ref: (28)

64

181. Clinical Therapeutics/Obstetrics/Gynecology


Which of the following is a contraindication to prescribing oral hormonal contraceptives?
A. history of iron deficiency anemia
B. history of dysmenorrhea since age 15
C. positive family history of ovarian cancer
D. history of smoking in a patient older than age 35
Explanations
(u) A. Use of oral hormonal contraceptives in patients with iron deficiency anemia is beneficial in that menstrual
periods are shorter with resultant decrease in blood loss.
(u) B. Oral hormonal contraceptives are one of the acceptable treatments for primary dysmenorrhea as periods are
less painful due to suppression of ovulation.
(u) C. Oral hormonal contraceptives have been proven to reduce the risk of ovarian and endometrial cancer. Current
recommendations include use in patients with a high risk of ovarian cancer due to family history or BRCA gene
status.
(c) D. A history of smoking in a patient older than age 35 is considered one of the absolute contraindications to
prescribing oral hormonal contraceptives due to the increased risk of thromboembolic events.
Ref: (4)

182. Clinical Therapeutics/Neurology


A 28 year-old female presents with complaint of headaches for the past several months. They typically begin behind
her right eye, which she describes as a combination of stabbing and pressure. She states that noise bothers her, and
she experiences nausea but no vomiting. She has occasionally awakened with the pain. She admits to being under a
lot of stress. Aspirin or acetaminophen does not relieve the pain. Which of the following is the best choice for initial
treatment of the acute headache?
A. topiramate (Topamax)
B. gabapentin (Neurontin)
C. propranolol (Inderal)
D. sumatriptan (Imitrex)
Explanations
(u) A. Beta blockers, calcium channel blockers, antiepileptic drugs, and tricyclic antidepressants have been used for
the prevention of migraine headaches, but not for the acute treatment.
(u) B. See A for explanation.
(u) C. See A for explanation.
(c) D. 5HT-receptor agonists and ergot alkaloids are effective for the acute treatment of migraine headaches if
acetaminophen or nonsteroidal anti-inflammatory drugs are not effective.
Ref: (11)

183. Health Maintenance/Orthopedics/Rheumatology


A 43 year-old male with a history of a right medial meniscectomy and a strong family history of osteoarthritis presents
to the clinic for a routine physical exam. He states he is very active and runs 20-25 miles a week and competes
routinely in 5 km races. He is 5' 10" and 160 lbs, BP is 128/76 and P 72. His physical examination is unremarkable.
Which of the following would you recommend to this patient to delay the onset of osteoarthritis?
A. weight loss
B. corticosteroid injections
C. 1500 mg of calcium daily
D. consider swimming or biking instead of running
Explanations
(u) A. The patient's BMI is 23. Obesity is not a contributing factor for this patient.
(u) B. Corticosteroid injections may be used for symptomatic treatment but are not used for prevention.
(u) C. Calcium supplementation is indicated for osteoporosis prevention.
(c) D. Swimming and biking would promote excellent joint motion and muscle strength and void the high-impact of
competitive short-distance running.
Ref: (11)

65

184. History & Physical/Cardiology


A 22 year-old male received a stab wound in the chest an hour ago. The diagnosis of pericardial tamponade is
strongly supported by the presence of
A. pulmonary edema.
B. wide pulse pressure.
C. distended neck veins.
D. an early diastolic murmur.
Explanations
(u) A. Pulmonary edema may result with low output states as seen with myocardial contusions, but it is not strongly
suggestive of tamponade.
(u) B. Wide pulse pressure is seen in conditions of high stroke volume such as aortic insufficiency or hyperthyroidism.
Narrow pulse pressure is seen with cardiac tamponade.
(c) C. Cardiac tamponade will manifest with distended neck veins and cold clammy skin.
(u) D. The onset of diastolic murmur is suggestive of valvular disease, not tamponade.
Ref: (11)

185. Clinical Intervention/Cardiology


A 45 year-old female presents with complaint of lower extremity discomfort. The patient admits to dull aching of the
left lower extremity. The discomfort is worse after standing for long periods of time. Examination reveals dilated,
tortuous and elongated veins on the medial aspect of the left leg. Pedal pulses are +2/4 bilaterally. There are no skin
changes or lower extremity edema noted. Which of the following is the most appropriate initial treatment of choice in
this patient?
A. heparin
B. compression stockings
C. furosemide (Lasix)
D. thrombectomy
Explanations
(u) A. Heparin is used in the treatment of deep vein thrombosis not varicose veins.
(c) B. This patient has signs and symptoms of varicose veins. Initial treatment with compression stockings may
prolong or avoid the need for surgery.
(u) C. This patient has no signs of edema or venous insufficiency requiring diuretic therapy.
(u) D. Thrombectomy is indicated in a patient with an arterial thrombus, this patient has intact pulses and no pallor.
Ref: (28)

186. Diagnosis/Cardiology
A 55 year-old male presents with complaint of sudden ripping chest pain that radiates into the abdomen. On
examination the patient is found to have diminished peripheral pulses and a diastolic murmur. EKG reveals left
ventricular hypertrophy. Which of the following is the most likely diagnosis?
A. acute myocardial infarction
B. pulmonary embolism
C. acute pericarditis
D. aortic dissection
Explanations
(u) A. Pain associated with a myocardial infarction is commonly a retrosternal pressure, squeezing, or heaviness. ST
segment elevation on EKG would be expected.
(u) B. A pulmonary embolism is associated with retrosternal pain; however chest pain is not always present. Patients
more commonly will have a sudden onset of dyspnea. PE is not usually associated with a diastolic murmur or
diminished pulses.
(u) C. Acute pericarditis is characterized by sharp, knife-like pain that is worse with lying supine and better with sitting
up and leaning forward. One would expect to find a pericardial friction rub on auscultation.
(c) D. Aortic dissection is characterized by a ripping or tearing type pain with radiation to the neck, back or abdomen.
Left ventricular hypertrophy is often seen on EKG secondary to longstanding hypertension. A diastolic murmur is
often present secondary to aortic insufficiency.
Ref: (28)

66

187. Diagnosis/ENT/Ophthalmology
A 2 year-old child is brought to the office because of a cough and a fever of 102 degrees F for 2 days. The physician
assistant notes the presence of hoarseness, a barking cough, and stridor. The ears and nose exam are
unremarkable. Auscultation of the chest reveals decreased breath sounds without crackles or expiratory wheezes.
Which of the following would be the initial diagnostic impression?
A. pneumonia
B. bronchiolitis
C. croup
D. asthma
Explanations
(u) A. Pneumonia would not produce the stridor noted on physical exam. Pneumonia would more than likely produce
crackles, also not noted in this patient.
(u) B. Bronchiolitis would produce inspiratory wheezes which are absent in this patient.
(c) C. Hoarseness, inspiratory stridor, and a barking cough are classic signs of croup, all of which are noted in this
patient.
(u) D. Asthma would not normally present with fever and stridor.
Ref: (29)

188. History & Physical/Dermatology


Which of the following skin rashes is pathognomonic of rheumatic fever?
A. erythema marginatum
B. erythema nodosum
C. diffuse petechiae
D. purulent bullae
Explanations
(c) A. Erythema marginatum is one of the major Jones criteria for the diagnosis of acute rheumatic fever.
(u) B. Erythema nodosum lesions are associated with many diverse causes including various infections,
malignancies, drug sensitivities and inflammatory/granulomatous diseases.
(u) C. Diffuse petechiae are most commonly seen in conditions that cause thrombocytopenia.
(u) D. Purulent bullae are noted in patients with scalded-skin syndrome.
Ref: (28)

189. History & Physical/Dermatology


Which of the following lesions is most characteristic of psoriasis?
A. hypopigmented macules on hands and forearms
B. pruritic pustules on finger webs and wrist creases
C. crusting vesicles around the mouth and face
D. scaling plaques on knees and elbows
Explanations
(u) A. Hypopigmented macules are more characteristic of vitiligo.
(u) B. Pruritic pustules are more characteristic of scabies.
(u) C. Crusting vesicles around the mouth and face are more characteristic of herpes or impetigo.
(c) D. Psoriasis most commonly presents with scaling plaques and papules on the scalp, elbows, forearms,
lumbosacral regions, knees, hands, and feet.
Ref: (10)

190. History & Physical/Orthopedics/Rheumatology


Which of the following clinical manifestations is commonly seen in a patient with rheumatoid arthritis?
A. symmetric joint swelling
B. presence of Heberden's nodes
C. morning stiffness lasting less than 15 minutes
D. cervical spondylosis of C3-C7

67

Explanations
(c) A. Symmetric joint swelling associated with stiffness, warmth, tenderness, and pain are characteristic of
rheumatoid arthritis.
(u) B. Heberden's nodes, or bony enlargements of the DIP joints, are seen in osteoarthritis, not rheumatoid arthritis.
(u) C. Morning stiffness lasting less than 15 minutes is more characteristic of osteoarthritis. The morning stiffness
typically seen with rheumatoid arthritis lasts longer than 1 hour and is a distinguishing feature between the two types
of arthritis.
(u) D. Cervical spondylosis occurs with osteoarthritis. RA is associated with C1-C2 subluxation.
Ref: (11)

191. History & Physical/Obstetrics/Gynecology


Which of the following is the most common presenting clinical manifestation of breast cancer?
A. breast tenderness
B. nipple discharge
C. nipple retraction
D. breast mass
Explanations
(u) A. The breast mass is usually painless; therefore, breast tenderness would not be present.
(u) B. While serous or bloody nipple discharge may be present, it is not the most common presenting clinical
manifestation of breast cancer.
(u) C. Nipple retraction is a later finding of breast cancer and indicates ductal involvement.
(c) D. A breast mass is the most common presenting clinical manifestation of breast cancer found by the patient or
health care provider.
Ref: (4)

192. Clinical Therapeutics/Urology/Renal


A 7 year-old boy wets the bed nearly every night. Which of the following is the best pharmaceutical agent to use in
treating this patient?
A. desmopressin (DDAVP)
B. paroxetine (Paxil)
C. lorazepam (Ativan)
D. hyoscyamine (Levsin)
Explanations
(c) A. Desmopressin, while not curative, will relieve symptoms.
(u) B. Paroxetine is not indicated for enuresis.
(u) C. Lorazepam is used for treatment of sleep terrors.
(u) D. Hyoscyamine is used in the treatment of pediatric overactive bladder.
Ref: (13)

193. Clinical Intervention/Gastrointestinal/Nutritional


The first step in the treatment of a patient with an intestinal obstruction and no comorbid diseases is
A. nasogastric decompression.
B. invasive hemodynamic monitoring.
C. abdominal exploration.
D. administration of antibiotics.
Explanations
(c) A. Nasogastric decompression is indicated in all but mild cases of obstruction to prevent distal passage of
swallowed air and minimize distension.
(u) B. Invasive hemodynamic monitoring is needed only if the patient has underlying cardiac, pulmonary, or renal
disease.
(u) C. Abdominal exploration is considered only if the obstruction does not resolve in 24 to 48 hours or if peritoneal
findings, fever, or rapidly progressing abdominal pain occur.
(u) D. Antibiotics are given only if surgery is to be done.
Ref: (30)

68

194. Diagnostic Studies/Gastrointestinal/Nutritional


A 65 year-old patient with known history of alcohol and tobacco abuse presents with solid food dysphagia. The
patient also has a 24 lb weight loss over the past 6 months. Which of the following is the most appropriate
intervention?
A. endoscopy with biopsy
B. chest x-ray
C. barium esophagogram
D. CT scan of the thorax
Explanations
(c) A. Endoscopy with biopsy establishes the diagnosis of esophageal cancer with a high degree of reliability.
(u) B. Chest x-ray may show adenopathy, a pulmonary or bony metastases or sign of tracheoesophageal fistula.
Barium esophagogram is obtained as the first study to evaluate the dysphagia. CT scan should be obtained once the
diagnosis of carcinoma has been made to evaluate for pulmonary or hepatic metastases, lymphadenopathy, and
local tumor extension. None of these tests will reveal the diagnosis of carcinoma.
(u) C. See B for explanation.
(u) D. See B for explanation.
Ref: (11)

195. Health Maintenance/Endocrinology


As part of the long-term management of a patient with type 1 diabetes mellitus, the glycosylated hemoglobin
(HgbA1C) level should be ideally maintained at
A. less than 5%.
B. 6 to 7%.
C. 8 to 10%.
D. greater than 10%.
Explanations
(u) A. See B for explanation.
(c) B. The HgbA1C in patients with type 1 diabetes mellitus should be maintained between 6 and 7%. Lower levels
lead to an increased number of episodes of hypoglycemia, and higher levels lead to an increased risk of retinopathy,
nephropathy, and neuropathy.
(u) C. See B for explanation.
(u) D. See B for explanation.
Ref: (28)

196. Health Maintenance/Obstetrics/Gynecology


An increased incidence of breast cancer is associated with
A. nulliparity.
B. late menarche.
C. trauma to the breast.
D. early natural menopause.
Explanations
(c) A. Nulliparous women have a 1.5 times higher incidence of breast cancer than multiparous women.
(u) B. Early, not late, menarche is associated with an increased risk of breast cancer.
(u) C. Previous trauma to the breast is not associated with an increased incidence of breast cancer.
(u) D. Late, not early, natural menopause is associated with an increased risk of breast cancer.
Ref: (28)

197. History & Physical/Cardiology


A 78 year-old patient who is in acute distress with near-syncope and lightheadedness is being examined. The
patient's pulse is 40/min and blood pressure is 90/56 mm Hg. Examination of the patient at 45 degrees of elevation
reveals cannon "a" waves. Which of the following is the most likely explanation for these abnormal waves?

69

A. atrioventricular dissociation
B. aortic stenosis
C. systolic hypertension
D. left ventricular hypertrophy
Explanations
(c) A. The patient is in a third-degree heart block with the atria contracting against a closed atrioventricular valve,
which would be the scenario in a patient who has an escape rate of 40. Elderly patients are at risk for heart
conduction problems such as complete heart block.
(u) B. Left ventricular hypertrophy, systolic hypertension, and aortic stenosis are not causes of "a" waves.
(u) C. See B for explanation.
(u) D. See B for explanation.
Ref: (11)

198. Clinical Therapeutics/ENT/Ophthalmology


A 19 year-old college student complains of a sore throat for over a week, with fever and general malaise. On exam T38 degrees C, P-70/minute, RR-20/minute, BP-110/76 mmHg. The skin is warm, dry, and without rash. The TMs
have a normal light reflex and the canals are clear. The oropharynx is inflamed, with bilaterally enlarged tonsils, and a
small amount of exudate. The neck is supple, with anterior cervical adenopathy. The abdomen is soft, nontender and
a spleen tip is palpable. The labs reveal a negative rapid strep screen and positive heterophil antibody. The WBC
count is 9,000/microliter with a differential of 40% atypical lymphocytes, 35% lymphocytes, 5% monocytes, 10%
eosinophils, and 10% neutrophils. Which of the following is the most appropriate treatment?
A. Penicillin
B. Erythromycin
C. Acetaminophen
D. Acyclovir
Explanations
(u) A. Antibiotics are not indicated in the treatment infectous mononucleosis, or Ebstein-Barr virus (EBV) infections.
(u) B. See A for explanation.
(c) C. Acetaminophen may be used to treat fever and pain associated with infectious mononucleosis, or EBV
infection.
(u) D. Acyclovir is not approved for use in treatment of EBV, although it is active against the EBV in vitro and in vivo.
It may be used in certain patients with AIDS, but has not been shown to affect the outcome of EBV in these patients.
Ref: (11)

199. Diagnosis/Gastrointestinal/Nutritional
A patient is hospitalized with a change in mental status. Examination reveals that he is unable to maintain dorsiflexion
of the wrists after pronating his arms in front of his body. Which of the following is the most likely diagnosis?
A. cocaine overdose
B. hyperthyroidism
C. hepatic encephalopathy
D. Parkinson's disease
Explanations
(u) A. Tremor and agitation are part of acute cocaine intoxication, not asterixis.
(u) B. Hyperthyroidism causes a fine resting tremor, not asterixis.
(c) C. This is the description for asterixis that is seen with hepatic encephalopathy, uremia, and carbon dioxide
narcosis.
(u) D. Parkinson's disease has resting tremor, rigidity, akinesia, and postural hypotension, not asterixis.
Ref: (28)

200. Diagnosis/Orthopedics/Rheumatology
A 20 year-old male presents with pain along the medial tibia. The pain initially began towards the end of soccer
practice but now it is present earlier on during practice. Physical exam reveals pain to palpation over the posterior
tibialis muscle body. What is the most likely diagnosis?

70

A. shin splint
B. stress fracture
C. Osgood-Schlatter disease
D. patellofemoral pain syndrome
Explanations
(c) A. Shin splints cause pain over the posterior tibialis muscle body as opposed to discrete pain over the tibia with a
stress fracture.
(u) B. See A for explanation.
(u) C. Osgood-Schlatter disease is an injury occurring at the insertion of the patellar tendon on the tibial tuberosity in
a younger age group.
(u) D. Patellofemoral pain syndrome is the most common cause of chronic anterior knee pain, more commonly seen
in females.
Ref: (5)

201. Diagnostic Studies/Pulmonology


Which of the following is the minimum criteria in millimeter diameter of induration for a positive tuberculosis skin-test
screening result in an HIV-positive patient?
A. 0
B. 5
C. 10
D. 15
Explanations
(u) A. See B for explanation.
(c) B. Current minimum criteria for a positive skin test is 5 mm in diameter for individuals at very high risk, including
those who are HIV infected and recent contacts.
(u) C. See B for explanation.
(u) D. See B for explanation.
Ref: (7)

202. Diagnostic Studies/Obstetrics/Gynecology


An 18 year-old female presents with 2-day duration of dysuria, vulvovaginal pruritis, and a frothy clear to white
discharge. Which of the following results would be expected?
A. clue cells on normal saline preparation
B. hyphae and budding yeast on a KOH preparation
C. intracellular gram negative diplococci on Gram stain
D. mobile flagellated protozoa on a normal saline preparation
Explanations
(u) A. Clue cells are seen in Gardnerella vaginalis infections, but the vaginal discharge is grayish and has an
unpleasant fishy odor.
(u) B. Candida infections are diagnosed by demonstrating hyphae and budding yeast on KOH and usually present
with a white cottage cheese discharge.
(u) C. Gonorrhea presents with a purulent discharge. Gram negative diplococci are not diagnostic in females.
(c) D. Trichomonas presents with a frothy discharge, irritative symptoms of pruritus, dysuria, and frequency, and the
flagellated protozoa are demonstrated on a saline preparation.
Ref: (8)

203. Clinical Intervention/Orthopedics/Rheumatology


A 36 year-old male has a history of recurrent low back pain. When lifting a stack of books yesterday, he experienced
sudden, severe pain in the lumbar area. He denies radicular pain. His lower extremity examination is unremarkable,
and his back examination is remarkable for paraspinal muscle tenderness and increased pain with flexion at the
waist. Which of the following management options should be instituted at this time?
A. refer the patient to orthopedic surgery
B. continue ordinary activities as tolerated

71

C. confine the patient to bed with traction


D. recommend sleeping on a softer mattress
Explanations
(u) A. The patient can be managed conservatively by a primary care provider since there are no neurological deficits.
(c) B. Continuation of activities as tolerated is recommended during the acute phase. After symptoms resolve, an
exercise program should be initiated to strengthen the back.
(u) C. Traction is an outdated method of treatment, while bed rest for more than a few days will cause muscle
atrophy.
(u) D. Management for low back strain includes using a firm mattress.
Ref: (11)

204. Clinical Therapeutics/Obstetrics/Gynecology


In which of the following maternal-fetal blood type pairings should the mother receive Rho-GAM?
A. A positive mother, O negative infant
B. A negative mother, O negative infant
C. AB positive mother, spontaneous abortion
D. AB negative mother, spontaneous abortion
Explanations
(u) A. See D for explanation.
(u) B. See D for explanation.
(u) C. See D for explanation.
(c) D. Rho-GAM is indicated for an unsensitized Rh-negative patient who has had a spontaneous or induced abortion,
ectopic pregnancy, or at the time of amniocentesis. It is also indicated at 28 weeks gestation and within 72 hours of
delivery of an Rh-positive infant.
Ref: (4)

205. Scientific Concepts/Neurology


Which of the following animals is the major cause of human rabies in the United States and, therefore, poses the
highest risk?
A. bats
B. rabbits
C. rodents
D. dogs
Explanations
(c) A. Bat rabies is found in practically every state and is the most common cause of human rabies in the U.S.
(u) B. Rabbits, rodents, and dogs are uncommon causes of human rabies in the United States.
(u) C. See B for explanation.
(u) D. See B for explanation.
Ref: (29)

206. Clinical Therapeutics/Psychiatry/Behavioral Medicine


A 33 year-old hypertensive patient presents with increasing feelings of worthlessness and hopelessness over the
past several months. Other symptoms include frequent crying episodes, loss of appetite, weight loss, insomnia with
daytime sleepiness, and thoughts of suicide without any specific plans in place at this time. Which of the following
would be the most appropriate first-line medication for this patient?
A. amitriptyline (Elavil)
B. bupropion (Wellbutrin)
C. fluoxetine (Prozac)
D. phenelzine (Nardil)

72

Explanations
(u) A. Tricyclic antidepressants, such as amitriptyline, are third-line drugs for treating major depression, and are
reserved for complicated and/or unresponsive cases.
(u) B. Bupropion is a second-line drug for treating major depression and should be avoided in patients with
hypertension.
(c) C. Selective serotonin reuptake inhibitors are the drugs of first choice for treating major depression.
(u) D. Phenelzine, a monoamine oxidase inhibitor, is reserved for treatment failures with supervision by a psychiatrist.
Ref: (14)

207. Scientific Concepts/ENT/Ophthalmology


Closure of the eyelids is mediated by which cranial nerve?
A. cranial nerve III
B. cranial nerve V
C. cranial nerve VII
D. cranial nerve IX
Explanations
(u) A. Cranial nerve III is involved in extraocular movement and controls opening of the eyelids but has no control
over closing the eyelids.
(u) B. Cranial nerve V controls the motor function of the temporal and masseter muscles and facial sensation. It has
no control over eyelid closure.
(c) C. Cranial nerve VII controls the motor function of the facial muscles not controlled by cranial nerve V (forehead,
eyebrows, mouth, and lips) including closing of the eyelids.
(u) D. Cranial nerve IX controls swallowing and the voice.
Ref: (3)

208. Scientific Concepts/Cardiology


Which of the following occurs as a result of pulmonary hypertension?
A. left atrial enlargement
B. aortic stenosis
C. coronary artery spasm
D. right ventricular enlargement
Explanations
(u) A. See D for explanation.
(u) B. See D for explanation.
(u) C. See D for explanation.
(c) D. In pulmonary hypertension increased resistance within pulmonary circulation causes the right ventricle to work
harder and eventually enlarge in response. Other changes that may occur are right atrial enlargement, decreased left
ventricular cavity size, and tricuspid regurgitation.
Ref: (7)

209. Clinical Intervention/Hematology


A 26 year-old male presents with a four week history of fatigue, night sweats, and a painless mass in his neck.
Physical examination confirms the presence of an enlarged right posterior cervical lymph node. What is the next best
step in the evaluation of this patient?
A. bone marrow biopsy and aspiration
B. lymph node biopsy
C. chest CT scan
D. reexamine in 2-4 weeks
Explanations
(u) A. See B for explanation.
(c) B. This patient presents with possible lymphoma. Diagnosis is made by lymph node biopsy. Bone marrow biopsy
and CT scan of the chest are used for staging of the disease.
(u) C. See B for explanation.
(u) D. With the presence of B-symptoms (fever, night sweats, and weight loss) the patient needs a biopsy to evaluate
for possible lymphoma.
Ref: (28)

73

210. Diagnosis/Obstetrics/Gynecology
A 13 year-old female presents with a six month history of lower mid-abdominal pain that is spasmodic in nature and
radiates to the inner thighs. The pain usually starts within a few hours of the onset of menses and lasts about 2 days.
The patient's menarche began 2 years ago. She denies any sexual activity. Physical examination, including pelvic, is
unremarkable. Which of the following is the most likely diagnosis?
A. ovarian cyst
B. endometriosis
C. primary dysmenorrhea
D. premenstrual syndrome
Explanations
(u) A. Most ovarian cysts are asymptomatic and found as an incidental finding on physical examination. Symptomatic
cysts usually present with acute pain or bleeding secondary to rupture.
(u) B. Endometriosis is characterized by pain that usually starts 1-2 weeks before the onset of menses and is relieved
at the onset of menstrual flow or shortly after.
(c) C. This patient most likely has primary dysmenorrhea supported by onset of pain within 1-2 years after onset of
menarche and characteristic symptoms of low, mid-abdominal, spasmodic cramping pain that radiates to the back or
inner thighs beginning on the first or second day of menstruation. Pelvic examination fails to reveal any pathological
findings.
(u) D. Premenstrual syndrome is seen primarily in females that are 25-40 years old. Associated physical and
emotional symptoms appear 7-14 days before the onset of menses and resolve once menstruation begins.
Ref: (4)

211. Clinical Therapeutics/Pulmonology


Which of the following is the most appropriate therapeutic agent for acute influenza?
A. azithromycin (Zithromax)
B. acyclovir (Zovirax)
C. tetracycline (Sumycin)
D. zanamivir (Relenza)
Explanations
(u) A. See D for explanation.
(u) B. See D for explanation.
(u) C. See D for explanation.
(c) D. Zanamivir is an anti-viral agent that is active against the influenza virus.
Ref: (28)

212. Clinical Intervention/Infectious Diseases


A 23 year-old female presents with a 24-hour history of watery diarrhea, nausea and vomiting after returning from a
cruise. On exam, the vitals reveal HR 120, BP 90/60, and RR 20. The abdomen is soft and non-tender, with
hyperactive bowel sounds. Which of the following is the most appropriate management at this time?
A. loperamide (Imodium)
B. IV hydration
C. bismuth sulfate (Pepto bismol)
D. ciprofloxacin (Cipro)
Explanations
(u) A. Loperamide is not indicated for acute infectious gastroenteritis in the vomiting patient.
(c) B. This is most likely a self-limiting viral gastroenteritis and is treated with hydration and supportive care.
(u) C. Bismuth sulfate is useful for treating diarrhea but is not helpful while the patient is vomiting.
(u) D. Antibiotics, like ciprofloxacin, are not indicated for viral infections.
Ref: (7)

213. Clinical Intervention/Urology/Renal


Which of the following is the treatment of choice for a nursing home patient who has asymptomatic bacteriuria with no
history of diabetes or structural abnormalities of the genitourinary tract?

74

A. ciprofloxacin (Cipro)
B. sulfamethoxazole-trimethoprim (Bactrim)
C. cephalexin (Keflex)
D. no treatment is needed
Explanations
(u) A. See D for explanation.
(u) B. See D for explanation.
(u) C. See D for explanation.
(c) D. Asymptomatic bacteriuria is commonly seen in the geriatric population and no treatment is needed as long as
the patient is not diabetic or has no structural abnormalities of the genitourinary tract.
Ref: (1)

214. Diagnostic Studies/Cardiology


A 56 year-old male presents to the office with a history of abdominal aortic aneurysm. He was told that he will need
ongoing evaluation to assess whether the aneurysm is expanding. What is the recommended study to utilize in this
situation?
A. plain film of the abdomen
B. serial abdominal exam
C. ultrasound of the abdomen
D. angiography of the abdominal aorta
Explanations
(u) A. Although some abdominal aortic aneurysms are calcified, abdominal radiography may demonstrate the
calcified outline of the aneurysm. However, about 25% of aneurysms are not calcified and cannot be visualized by
plain x-ray.
(u) B. Serial abdominal exams are not sensitive in detecting progression of abdominal aortic aneurysms.
(c) C. An abdominal ultrasound can delineate the transverse and longitudinal dimensions of an abdominal aortic
aneurysm and may detect mural thrombus. Abdominal ultrasound is best used to screen patients at risk for the
development of this condition.
(u) D. Contrast aortography is used commonly for the evaluation of patients with aneurysms before surgery, but it has
no role in the serial assessment of patients being followed on a chronic basis.
Ref: (11)

215. Scientific Concepts/Hematology


Absorption of vitamin B12 requires intrinsic factor that is secreted by which of the following?
A. parietal cells
B. chief cells
C. mucous cells
D. enteroendocrine cells
Explanations
(c) A. Parietal cells are responsible for acid secretion and intrinsic factor.
(u) B. Chief cells are responsible for the secretion of the proteolytic proenzymes pepsinogen I and II.
(u) C. Mucous cells secrete mucous and pepsinogen II.
(u) D. Enteroendocrine cell secrete gastrin.
Ref: (7)

216. Clinical Therapeutics/Orthopedics/Rheumatology


A 35 year-old patient with lupus is being treated for mild arthralgias and rash with hydroxychloroquine. Which of the
following clinical manifestations is the most common side effect of this medication?
A. impaired night vision
B. jaundice
C. proteinuria
D. mouth sores

75

Explanations
(c) A. Hydroxychloroquine is associated with macular damage, rash and diarrhea.
(u) B. Hydroxychloroquine is not associated with hepatoxicity of hemolytic anemia.
(u) C. Hydroxychloroquine lacks renal toxicity.
(u) D. Mouth sores may be a sign of agranulocytosis in patients on hydroxychloroquine, but retinopathy occurs more
commonly.
Ref: (9)

217. Diagnosis/Cardiology
A 55 year-old morbidly obese male is seen in the office for routine examination. He has a history of pulmonary
hypertension and cor pulmonale. Examination reveals a visible jugular venous pulse and a systolic flow murmur on
the right side of the sternum. Which of the following is the most likely diagnosis?
A. mitral insufficiency
B. tricuspid insufficiency
C. hepatic vein thrombosis
D. aneurysm of the thoracic aorta
Explanations
(u) A. Mitral insufficiency results in the accumulation of blood primarily in the pulmonary system and not the right side
of the heart.
(c) B. Tricuspid insufficiency will result in blood being put back into the right side of the body with increased jugular
pulsation in the neck, along with a palpable venous pulse in the liver.
(u) C. Hepatic vein thrombosis or Budd-Chiari syndrome is associated with cirrhosis and liver clotting abnormalities
and is not due to right-sided heart failure.
(u) D. Thoracic aorta aneurysm results in a widened mediastinum that is fairly asymptomatic until it results in rupture
or dissection. These are typically found as incidental findings unless they are symptomatic from dissection or rupture,
which causes severe chest pain or a severe tearing sensation in the chest.
Ref: (11)

218. Diagnosis/Psychiatry/Behavioral Medicine


A 56 year-old man is admitted to a hospital unit for evaluation of rectal bleeding and weight loss. He has a strong
family history of cancer. Soon after admission, a barium enema is scheduled. The patient refuses the "prep" because
he fears x-ray radiation. He states he has had previous x-rays, but becomes frightened at the thought of an x-ray and
"can't face it." The most likely diagnosis is
A. hysterical personality.
B. dissociative state.
C. conversion reaction.
D. phobic neurosis.
Explanations
(u) A. Hysterical personality consists of multiple physical complaints referable to several other organ systems.
(u) B. Dissociative state is precipitated by an emotional event that produces fugue, amnesia, somnambulism, multiple
personality, and depersonalization.
(u) C. Conversion reaction is characterized by physical symptoms in parts of the body related to psychic conflict.
(c) D. Phobic neurosis is a phobic ideation of displacement where the patient transfers feelings of anxiety from the
object to one that can be avoided.
Ref: (14)

219. Clinical Intervention/Cardiology


A 68 year-old patient presents after a syncopal episode. The patient has a history of coronary artery disease and
ischemic cardiomyopathy. Echocardiogram shows an ejection fraction of 20%. Electrophysiology study reveals
inducible sustained ventricular tachycardia from the left ventricle. Which of the following is the most appropriate
therapy in this patient?
A. implantable defibrillator
B. metoprolol (Lopressor)
C. radiofrequency ablation
D. warfarin (Coumadin)

76

Explanations
(c) A. Patients with symptomatic ventricular tachycardia (VT) or sustained VT and left ventricular dysfunction are at
increased risk for sudden cardiac death. An implantable defibrillator is the treatment of choice.
(u) B. Beta blockers are used in patients with nonsustained VT and normal ventricular function. They may be used as
an adjunct to, but not in place of, implantable defibrillator therapy in patients with symptomatic VT or sustained VT.
(u) C. Radiofrequency ablation is indicated in patients with outflow tract or fascicular tachycardia, not left sided VT.
(u) D. Anticoagulation therapy is indicated in patients with atrial fibrillation not VT.
Ref: (28)

220. Scientific Concepts/Cardiology


Which of the following is the most common cause of infective endocarditis in an IV drug abuser?
A. Haemophilus parainfluenza
B. Enterococci
C. Staphylococcus aureus
D. Viridans streptococci
Explanations
(u) A. See C for explanation.
(u) B. See C for explanation.
(c) C. S. aureus accounts for more than 60% of all cases of endocarditis in IV drug abusers.
(u) D. See C for explanation.
Ref: (28)

221. Diagnosis/Gastrointestinal/Nutritional
A 3 year-old presents with a 24-hour history of diarrhea. The patient is afebrile and the stool is noted to be loose and
watery. No blood is noted in the stool. Fecal WBC is negative. Which of the following is the most likely diagnosis?
A. viral gastroenteritis
B. toxic megacolon
C. ulcerative colitis
D. intussusception
Explanations
(c) A. Patients with viral gastroenteritis are afebrile and noted to have loose, watery, non-bloody diarrhea. Fecal WBC
is negative.
(u) B. Toxic megacolon is a complication of ulcerative colitis and presents with diarrhea, fever, tachycardia, and
leukocytosis.
(u) C. Ulcerative colitis presents with bloody diarrhea, abdominal pain, and fever.
(u) D. Intussusception is more common in children age 1-2 years and presents with crampy abdominal pain, vomiting,
and currant jelly stool.
Ref: (5)

222. History & Physical/Cardiology


Which of the following is an expected finding in a patient with a diagnosis of an arterial embolism?
A. lower extremity edema
B. stasis dermatitis
C. palpable cord
D. pulselessness
Explanations
(u) A. Lower extremity edema is commonly associated with venous insufficiency, not arterial embolism.
(u) B. Stasis dermatitis is commonly seen in patients with venous insufficiency, not arterial embolism.
(u) C. A palpable cord is commonly found in a patient with a DVT, not arterial embolism.
(c) D. Pulselessness is a sign of acute ischemia secondary to arterial embolism.
Ref: (28)

77

223. Diagnosis/Gastrointestinal/Nutritional
A 16 day-old male presents in the office with a history of vomiting after feeding for the past 2 days. The vomiting has
become progressively worse and the mother describes it as very forceful, sometimes hitting the floor 6 feet away.
She says the neonate is always hungry. On physical examination, it is noted that he is 2 ounces below birth weight,
and has a small palpable mass (about 1.5 cm) in the epigastrium. The most likely diagnosis is
A. achalasia.
B. tracheoesophageal fistula.
C. pyloric stenosis.
D. Meckel's diverticulum.
Explanations
(u) A. Achalasia is uncommon under the age of 5 and the child presents with retrosternal pain and dysphagia.
(u) B. Tracheoesophageal fistula presents with increased secretions, choking, cyanosis, and respiratory distress
within the first few hours of life.
(c) C. Pyloric stenosis begins between 2 to 4 weeks of age with vomiting that becomes projectile after each feeding.
An olive-size mass can often be felt in the epigastrium.
(u) D. Meckel's diverticulum presents with painless rectal bleeding. Vomiting is rare unless obstruction has occurred.
Ref. (5)

224. Diagnosis/Endocrinology
A 72 year-old female is being evaluated for recurrent kidney stones. Physical examination reveals no abnormal
findings. Laboratory findings show elevated calcium and decreased phosphate levels. Which of the following is the
most likely diagnosis?
A. pheochromocytoma
B. adrenal insufficiency
C. hyperparathyroidism
D. vitamin D deficiency
Explanations
(u) A. Pheochromocytoma may lead to hypercalcemia but the patient does not have any signs or symptoms
suggestive of pheochromocytoma, such as hypertension, headache, profuse sweating, or weight loss.
(u) B. Adrenal insufficiency, Addison's disease, would reveal, in addition to the hypercalcemia, anorexia, nausea and
vomiting, weight loss, and cutaneous hyperpigmentation, none of which are evident in this patient.
(c) C. The majority of patients with hyperparathyroidism are asymptomatic. Recurrent nephrolithiasis may be one of
the presentations of primary hyperparathyroidism. Measurement of parathyroid levels would be the initial laboratory
test for the evaluation of hypercalcemia.
(a) D. Vitamin D deficiency leads to hypocalcemia.
Ref: (11)

225. History & Physical/Hematology


A 55 year-old non-smoking male presents with a hemoglobin of 18.5 g/dl and a hematocrit of 56%. Which of the
following physical examination findings is the most likely to be noted with this patient?
A. splenomegaly
B. cheilosis
C. purpura
D. decreased vibratory sense
Explanations
(c) A. Patients with polycythemia vera present with elevated hemoglobin and hematocrit. On physical examination
plethora, engorged retinal veins, and splenomegaly are common.
(u) B. Cheilosis is noted in iron deficiency anemia.
(u) C. Purpura is typically noted in bleeding disorders.
(u) D. Decreased vibratory sense is noted in vitamin B12 deficiency.
Ref: (28)

78

Association of Physician Assistant Programs


PACKRAT

Comment Form
Please fill out this form and return it to: Physician Assistant Education Association
300 N. Washington St., Ste 505
Alexandria, VA 22314
YES

NO

1. I found the PACKRAT a useful study tool.


Comment:

2. I was able to understand the directions in the Directions and Explanations.

Comment:

3. The feedback provided was easy to understand.


Comment:

4. I know where my weaknesses lie and where to focus my studies.


Comment:

5. The content of the examination was comprehensive.


Comment:

79

6. The examination questions were clearly worded.


Comment:

7. Suggestions for improvement:


_________________________________________________________________
_________________________________________________________________

80

Vous aimerez peut-être aussi